Вы находитесь на странице: 1из 72

Uveitis

Question 1 of 130
A 27-year-old man presents with a history of floaters and blurred vision OD. The patient immigrated to the United States
from India within the last year, is currently in good health, and denies any history of drug use. Two years ago, while in India,
he had a positive tuberculin skin test, and received 9 months of prophylactic isoniazid, rifampin, and ethambutal therapy.
Anterior segment examination is normal. The right fundus is shown in the figure. The left fundus showed findings similar to
those of the right. Vitreous hemorrhage is noted in the inferior vitreous OD. No pars plana snowbanks are noted in either
eye. Which of the following would be least helpful in the workup of this patient?

Serum glucose

Angiotensin-converting enzyme (ACE) and antinuclear antibody (ANA) titers

Hemoglobin electrophoresis

HLA-A29
Please select an answer
Feedback: This patient has bilateral retinal neovascularization, which may arise from a number of conditions, including
diabetes mellitus, branch retinal vein occlusion (BRVO), sickle-cell retinopathy, Eales' disease, or from inflammatory
conditions such as sarcoidosis, tuberculosis (TB), Behcet's disease, peripheral uveitis, or systemic lupus erythematosus
(SLE). Workup should be directed toward these diagnoses and should include a serum glucose and/or glucose tolerance
test, fluorescein angiography (for BRVO), hemoglobin electrophoresis (for sickle-cell retinopathy), chest xray (for TB,
sarcoid), ACE titer (for sarcoid), ANA titer (for SLE), HLA-B5 (for Behcet's disease), and a tuberculin skin test with anergy
panel (for TB and sarcoid). HLA-A29 would not need to be tested in this patient, because it is associated with birdshot
retinochoroidopathy, which produces a retinochoroiditis, but not retinal neovascularization.
Question 2 of 130
A patient presents with meningismus, vitiligo, and bilateral exudative retinal detachments. What diagnosis is most consistent
with these findings?

Herpes simplex uveitis

Human leukocyte antigen (HLA)-B27 uveitis

Vogt-Koyanagi-Harada syndrome

Fuchs heterochromic iridocyclitis


Please select an answer
Feedback: Vogt-Koyanagi-Harada syndrome is a multi-system disorder featuring ocular inflammation (often with exudative
detachments), CNS findings, such as meningismus, and auditory or vestibular findings. Later in the diesease course, skin is
involved including vitiligo, poliosis, and vitiligo. It is always bilateral, wherase herpes simplex, Fuchs heterochromic cyclitis,
and HLA-B27 uveitis more typically present unilaterallyl
Question 3 of 130
A 27-year-old man presents with a history of floaters and blurred vision OD. The patient immigrated to the United States
from India within the last year, is currently in good health, and denies any history of drug use. Two years ago, while in India,
he had a positive tuberculin skin test and received 9 months of prophylactic isoniazid, rifampin, and ethambutal therapy.
Anterior segment examination is normal. The right fundus is shown. The left fundus showed findings similar to those of the
right. Vitreous hemorrhage is noted in the inferior vitreous OD. No pars plana snowbanks are noted in either eye. The
fluorescein angiogram of the right eye of the patient is also shown. Extensive retinal nonperfusion, without capillary
telangiectasia or collateral formation, is present in the inferior retina of each eye. If laboratory investigation of the patient
shows a normal serum glucose, normal hemoglobin electrophoresis, normal angiotensin-converting enzyme, normal chest x-
ray, and negative HLA-B5 and antinuclear antibody titer, which of the following statements would be most accurate?
The patient's findings are most consistent with bilateral branch retinal vein occlusion, and further workup for an
underlying hematologic abnormality should be performed.

The patient's laboratory and retinal findings support the diagnosis of Eales' disease.

Unlike with proliferative diabetic retinopathy, in which neovascularization of the iris may develop, this patient has little
chance of ever developing iris neovascularization.

The retinal neovascularization in this patient is likely to remain stable; therefore, treatment is not indicated.
Please select an answer
Feedback: Eales' disease produces an obliterative perivasculitis that leads to retinal ischemia, retinal neovascularization,
vitreous hemorrhage, and, later, tractional retinal detachment and iris neovascularization. Retinal nonperfusion is seen in
proliferative diabetic retinopathy, in which neovascularization of the optic disc, retina, and iris may occur. Retinal
nonperfusion can also be seen in primary branch retinal vein occlusion (BRVO), and can lead to secondary retinal
neovascularization. The angiogram in this case, however, does not show capillary telangiectasia, collateral vessels, or a
pattern of involvement along the distribution of a particular vein, which are findings typically seen in BRVO. This patient's
young age, gender, ethnic background, and positive tuberculin status are characteristics commonly associated with Eales'
disease. However, there are no diagnostic tests for Eales' disease, and this diagnosis can be made only when other causes
have been excluded. The laboratory data for this patient rule against diabetes mellitus, hemoglobinopathies, sarcoidosis,
Behcet's, and systemic lupus erythematosus as underlying causes of the retinal findings. Retinal neovascularization in
Eales' disease is observed in up to 80% of patients and frequently leads to vitreous hemorrhage, and less commonly, to
retinal detachment. Laser photocoagulation of areas of peripheral retinal ischemia may produce regression of retinal
neovascularization.
Question 4 of 130
A 73-year-old man complains of increasing pain and decreasing vision OD over the last 3 days. Two weeks ago he
underwent an uncomplicated phacoemulsification, and placement of a foldable posterior chamber silicone intraocular lens
(IOL) in the same eye. On examination, his visual acuity is HM 00 and 20/30 OS. Intraocular pressures are normal. Slitlamp
examination OD reveals moderate conjunctival injection, a Seidel negative cataract wound, a 1.5-mm hypopyon in the
anterior chamber, and a fibrin plaque over the anterior surface of the IOL. A dull red reflex is present, and fundus details are
not visible. Ultrasonography shows diffuse vitreous debris, but no retinal or choroidal detachment. The patient's findings are
consistent with postoperative endophthalmitis. Based on the Endophthalmitis Vitrectomy Study (EVS) findings, what would
be the recommended initial course of treatment for this patient?

Immediate pars plana vitrectomy and injection of intravitreal antibiotics

Intravenous antibiotics, immediate pars plana vitrectomy, and injection of intravitreal antibiotics

Vitreous tap and injection of intravitreal antibiotics

Intravenous antibiotics, vitreous tap, and injection of intravitreal antibiotics


Please select an answer
Feedback: The EVS was a prospective, randomized clinical trial that examined the role of intravenous antibiotics and
immediate pars plana vitrectomy in the treatment of postoperative endophthalmitis occurring within 6 weeks of cataract
extraction or placement of a secondary intraocular lens. All patients received intravitreal antibiotics. Patients were
randomized into four groups based on whether or not they received intravenous antibiotics, or underwent immediate pars
plana vitrectomy vs. intravitreal tap (PPV vs TAP). The results demonstrated no difference in final visual acuity or media
clarity with the use of systemic antibiotics. Patients with an initial visual acuity of HM or better showed no difference in final
visual acuity whether or not immediate pars plana vitrectomy was performed. However, patients with an initial visual acuity
of light perception (LP) were found to benefit from immediate vitrectomy. Patients with LP vision who received immediate
vitrectomy were found to have a three-fold increase in achieving 20/40 or better (33% PPV vs 11% TAP), a two-fold increase
in achieving 20/100 or better (56% PPV vs 30% TAP), and a 50% decrease in the frequency of severe visual loss, defined
as <5/200 (20% PPV vs 47% TAP). Therefore, based on the EVS findings, this patient does not require intravenous
antibiotics or immediate vitrectomy, because his presenting visual acuity is HM.
Question 5 of 130
A 27-year-old man presents with complaints of painless, progressive distortion, and blurring of vision OD over the last year.
He previously noted floaters OD periodically over many years. He has no known medical problems, no history of premature
birth, and no history of prior drug use. There is no family history of visual problems. On examination, visual acuity is 20/80
OD, 20/20 OS, no injection is present, anterior chambers are quiet OU, and lenses are clear. The anterior vitreous has trace
cell OD, no cell OS. The right fundus is shown in the figures (temporal periphery, A; posterior pole, B). The left fundus
examination was normal. Which one of the following is the most likely diagnosis in this patient?
Peripheral granuloma from Toxocara canis

Coats' disease

Toxoplasmosis

Familial exudative vitreoretinopathy


Please select an answer
Feedback: This patient presents with a unilateral, peripheral subretinal mass, with numerous epiretinal membranes
extending from the peripheral lesion toward the posterior pole. His symptoms of metamorphopsia and blurred vision are
likely the result of the epiretinal membrane producing macular distortion/traction. The peripheral lesion is most consistent
with an old peripheral granuloma from Toxocara canis. Ocular involvement with Toxocara canis typically manifests itself
without pain or redness, and is unilateral. Toxocara canis may present as a posterior pole granuloma, endophthalmitis, or a
peripheral granuloma. The patient's previous symptom of periodic floaters may relate to a time when this lesion was more
inflamed. Coats' disease in this patient is less likely, because intraretinal telangiectatic vessels and subretinal exudate are
not present. Familial exudative vitreoretinopathy is typically bilateral, and a family history would be anticipated.
Toxoplasmosis, although unilateral, shows much more retinal and vitreous inflammation. Also, inactive toxoplasmic lesions
appear different from this lesion in that they usually result in an atrophic chorioretinal scar, rather than a subretinal mass.
Question 6 of 130
A 35-year-old woman from the Midwest region of the United States experienced loss of vision in the right eye to a level of
20/400. Examination revealed multiple discrete choroidal scars. The vision subsequently improved to 20/30, but a relapse
occurred in which the level of vision in the right eye declined to counting fingers. The funduscopic findings in the right eye
are illustrated in the figure. Cells were present in the posterior vitreous. Fluorescein angiography revealed staining of the
RPE lesions in the right eye. Which of the following clinical entities is most consistent with this clinical presentation?
Vitiliginous (birdshot) retinochoroiditis

Acute multifocal placoid pigment epitheliopathy

Multifocal choroiditis

Presumed ocular histoplasmosis syndrome


Please select an answer
Feedback: This clinical presentation is most consistent with multifocal choroiditis, a disorder in which loss of vision is
accompanied by the appearance of yellow-white lesions in the posterior pole at the level of the RPE and choroid (see figure
1). These lesions eventually change into atrophic chorioretinal scars. This condition differs from the presumed ocular
histoplasmosis syndrome (POBS; figure A ) in that there is a female sex predilection, the fundus lesions are smaller, there is
rarely peripapillary chorioretinal atrophy, anterior chamber inflammation is present in 50% or more of the affected subjects,
and vitreous cells are present in all patients. Multifocal choroiditis is similar to POHS, because both may be complicated by
secondary choroidal neovascularization (CNV), which may severely decrease visual acuity if it involves the macula.
Vitiliginous retinochoroiditis afflicts an older subset of patients and results in millet seed-like yellow lesions at the level of the
RPE, typically present outside the posterior pole (figure B). Vitreous cells and cystoid macular edema may also be found in
patients with vitiliginous retinochoroiditis. AMPPE is typically a bilateral condition characterized by larger, cream-colored
lesions at the level of the RPE (figure C).
Question 7 of 130
A 30-year-old woman with a history of sarcoidosis presents with the retinal finding shown in the figure. Which of the following
statements is least accurate regarding this patient?

This patient is likely to demonstrate hilar lymphadenopathy and/or parenchymal infiltrates on chest x-ray.

This patient may not demonstrate anergy on tuberculin skin testing.

Retinal neovascularization, which may accompany this condition, typically does not respond to systemic corticosteroids
or scatter photocoagulation.

This patient has an increased risk of central nervous system involvement.


Please select an answer
Feedback: Sarcoidosis is a multisystem, granulomatous disease that most commonly affects the lungs, skin, lymph nodes,
and eyes. Pulmonary involvement, which may involve the lung parenchyma or hilar lymph nodes, may affect 90% of patients
with sarcoid. Therefore, a chest x-ray may be helpful in detecting patients with the disease. Other laboratory tests that may
be helpful in making the diagnosis include serum angiotensin-converting enzyme levels, which may be elevated in 60%-90%
of patients; skin testing, which may demonstrate anergy in 50% of patients; and whole-body gallium scanning, which may
demonstrate uptake in areas of active inflammation. Ocular involvement may be seen in 33%-50% of patients with sarcoid.
Patients with ocular involvement most often present with a granulomatous, anterior uveitis, but may have a myriad of
manifestations, including cataract, glaucoma, dry eye, conjunctival nodules, lacrimal gland enlargement, vitritis, retinal
periphlebitis, chorioretinitis, cystoid macular edema, peripheral uveitis, optic disc edema, and retinal neovascularization.
Patients who have posterior uveitis are believed to have a higher incidence of central nervous system involvement (35%)
than patients without (5%). Topical corticosteroids are usually effective in treating the anterior uveitis from sarcoidosis, but
posterior disease usually requires periocular or systemic corticosteroids. Retinal neovascularization secondary to sar-
coidosis has also been shown to respond to systemic corticosteroids. Retinal neovascularization secondary to retinal
ischemia may respond to scatter laser photocoagulation.
Question 8 of 130
A 50-year-old woman presents with decreased visual acuity in her right eye of 2 weeks duration. Visual acuity is 20/200 OD
and 20/20 OS. Anterior segment examination of the right eye reveals keratitic precipitates on the cornea, moderate cell and
flare, normal iris, and mild nuclear sclerosis. The left anterior segment examination is unremarkable, except for mild nuclear
sclerosis. Anterior and posterior vitreous cells are noted in the right eye. The vitreous and retina in the left eye are normal.
The fundus findings in the right eye are shown in the figures (posterior pole, A; temporal midperiphery, B). Fluorescein
angiographic findings in the temporal midperiphery are shown in part C of the figure. This patient was noted on recent chest
x-ray to have a cavitary lesion in the right upper lobe. Given these findings, which laboratory test would be least helpful in
working up this patient?
Antineutrophil cytoplasmic antibodies (ANCAs)

HLA-B27 typing

Angiotensin-converting enzyme (ACE)

Tuberculin skin testing with positive control


Please select an answer
Feedback: This patient presents with occlusive retinal vasculitis and granulomatous uveitis. The differential diagnosis
includes tuberculosis, sarcoidosis, Wegener's granulomatosis, Behçet's disease, herpetic retinal infection, and syphilis. In
view of the chest x-ray findings, Wegener's granulomatosis and tuberculosis should be most strongly considered.
Antineutrophil cytoplasmic antibodies (ANCAs) are specific markers for Wegener's granulomatosis. They have been shown
to be positive in approximately 90% of patients with biopsy-proven Wegener's granulomatosis, although this may vary with
disease activity. Tuberculin skin testing with a positive control may be helpful in diagnosing tuberculosis, or if anergy is
present, be indicative of sarcoid. In addition, it is important to know the patient's purified protein derivation (PPO) status,
because it is likely that corticosteroids or immunosuppressive agents may be required for treatment of the retinal vasculitis.
In addition to hilar lymphadenopathy or granulomata in the lung parenchyma on chest x-ray, increased uptake on gallium
scanning or an elevated serum ACE may be helpful in diagnosing sarcoidosis. HLAB27-associated uveitides (eg, ankylosing
spondylitis, Reiter's syndrome) are typically nongranulomatous and do not have an associated retinal vasculitis; therefore,
tissue typing for HLA-B27 would not be useful.

Question 9 of 130
A 73-year-old man complains of increasing pain and decreasing vision OD over the last 3 days. Two weeks ago he
underwent an uncomplicated phacoemulsification, and placement of a foldable posterior chamber silicone intraocular lens
(IOL) in the same eye. On examination, his visual acuity is HM 00 and 20/30 OS. Intraocular pressures are normal. Slitlamp
examination OD reveals moderate conjunctival injection, a Seidel negative cataract wound, a 1.5-mm hypopyon in the
anterior chamber, and a fibrin plaque over the anterior surface of the IOL. A dull red reflex is present, and fundus details are
not visible. Ultrasonography shows diffuse vitreous debris, but no retinal or choroidal detachment. The patient's findings are
consistent with postoperative endophthalmitis. The patient experienced decreasing inflammation and noted improved visual
acuity for the first 2 days following treatment. On the third post treatment day, however, visual acuity decreased and a new
vitreous hemorrhage was noted. Anterior segment examination demonstrated minimal corneal epithelial edema, mild cell
and flare in the anterior chamber, a residual fibrin plaque on the anterior surface of the IOL, which is centered, and an open
posterior capsule. Ultrasonography revealed a retinal detachment. Pars plana vitrectomy, air-fluid exchange, and laser
retinopexy are planned to repair the retinal detachment, because the vitreous hemorrhage prevents an adequate view for
scleral buckling. Which of the following conditions in this patient may present the most difficulty for completing the planned
treatment?

Residual fibrin plague on the anterior surface of the IOL

Presence of a silicone IOL

Vitreous hemorrhage

Previous intravitreal antibiotics


Please select an answer
Feedback: There is a risk of retinal detachment following pars plana vitrectomy or vitreous tap. This risk is increased in
patients with endophthalmitis, because the infection and resultant inflammation may induce areas of retinal weakening,
vitreous organization, and increased vitreoretinal adhesion. The vitreous hemorrhage in this patient is most likely the result
of a retinal break development. Visualization of the retina in this patient may be made difficult by epithelial corneal edema,
residual fibrin on the IOL, residual vitreous debris, and vitreous hemorrhage. However, the corneal epithelium may be
debrided, the fibrin plaque stripped from the IOL, and vitrectomy will remove the vitreous opacities such that none of these
should pose a significant obstacle in repairing the retinal detachment as planned. The use of prior intravitreal antibiotics
should not directly affect the performance of the vitrectomy. The patient's clinical response to the initial treatment for
endophthalmitis suggests the infection was adequately treated and may not require further intravitreal antibiotics, which
would not be possible with the use of intravitreal air. The presence of a silicone intraocular lens, however, may pose a signif-
icant problem with visualization during vitrectomy once an air-fluid exchange is performed. Frequent condensation like fluid
deposits have been noted to occur on the posterior surface of foldable silicone IOLs with open posterior capsules. This
condensation may severely impair the view of the retina during the air-fluid exchange and may be difficult to clear during
surgery. Obviously, endophthalmitis and postoperative retinal detachment cannot be anticipated, but the use of foldable
silicone IOLs should be carefully considered in patients who may require later retinal surgery.

Question 10 of 130
The ultrasonogram shown in the figure was obtained from an 18-year-old man who presented with complaints of ocular pain,
redness, mild proptosis, and blurred vision OD of 2 weeks duration. Which of the following would be least helpful in the
diagnostic workup of this patient?

Computerized tomography of the orbits

Rheumatoid factor

HLA typing for Behçet's disease

Antinuclear antibodies (ANA)


Please select an answer
Feedback: Because this patient presents with proptosis, redness, and pain, orbital causes, such as diffuse idiopathic orbital
inflammation (pseudotumor) and thyroid ophthalmopathy, must still be carefully considered despite the ultrasonographic
findings of a "T" sign. Therefore, computed tomography may be helpful to rule out orbital disease. In fact, idiopathic orbital
inflammation may be very difficult to discern from posterior scleritis in patients who present with proptosis, chemosis, and
limitation of ocular movements. Ultrasonography and computed tomography may show scleral thickening and retrobulbar
edema, in addition to extraocular muscle enlargement in either condition. Systemic disease has been found in 45% of
patients with posterior scleritis, including rheumatoid arthritis, systemic lupus erythematosus (SLE), psoriatic arthritis, giant
cell arteritis, and gout. Therefore, laboratory tests such as ANA (for SLE and rheumatoid arthritis), rheumatoid factor (for
rheumatoid arthritis), erythrocyte sedimentation rate (for giant cell arteritis), uric acid (for gout), and limb xrays (for
rheumatoid arthritis, psoriatic arthritis, and gout) are helpful. Behçet's disease has not been associated with posterior
scleritis and has only rarely been associated with anterior scleritis; therefore, HLA typing for Behçet's disease would be of
little value.

Question 11 of 130
The ultrasonogram shown in the figure was obtained from an 18-year-old man who presented with complaints of ocular pain,
redness, mild proptosis, and blurred vision OD of 2 weeks duration. Which of the following is the most likely diagnosis in this
patient?
Idiopathic uveal effusion syndrome

Thyroid ophthalmopathy

Vogt-Koyanagi-Harada (VKH) syndrome

Posterior scleritis
Please select an answer
Feedback: The ultrasound demonstrates the “T” sign that is characteristic for posterior scleritis. The ultrasound below has
an arrow added to point out the echolucent area, which is produced by retrobulbar edema sourrounding the optic nerve.
Frequently, the choroid and sclera are thickened in posterior scleritis, as shown in this patient. Occasionally, serous retinal
and choroidal detachments may also be present. Ultrasonography in idiopathic uveal effusion syndrome typically shows
thickened choroid and sclera, as well as serous choroidal and retinal detachments. Patients with this syndrome, however, do
not have retrobulbar edema, as in posterior scleritis, and typically do not present with pain or redness, which is common in
posterior scleritis. In addition, patients with idiopathic uveal effusion have a nanophthalmic eye rather than proptosis, which
may be seen in posterior scleritis. Thyroid ophthalmopathy may present with protosis, redness, and decreased vision, but
typically only demonstrates enlarged extraocular muscles on ultrasonography. VKH syndrome also does not demonstrate
retrobulbar edema on ultrasonography, but does typically show thickened choroid and serous retinal detachments.
Question 12 of 130
What does the differential diagnosis of idiopathic pars planitis include?

Ankylosing spondylitis

Rheumatoid arthritis

Lymphoma

Vogt-Koyanagi-Harada syndrome
Please select an answer
Feedback: Pars planitis is a form of intermediate uveitis featuring vitritis, pars plana snowbanks, and frequently cystoid
macular edema. It is bilateral in 90% of cases. Intraocular lymphoma can also present with bilateral vitritis, with or without a
yellowish subretinal infiltrate. Ankylosing spondylitis is associated with a recurrent acute unilateral anterior uveitis in most
cases. Rheumatoid arthritis is more commonly associated with scleritis and peripheral ulcerative keratitis. Vogt-Koyanagi-
Harada disease features bilateral granulomatous panuveitis, frequently with exudative retinal detachment, meningismus and
tinitus, and later in disease course, integument findings.

Question 13 of 130
Sterility is a side effect associated with which of the following immunomodulatory drugs?

Mycophenolate mofetil

Azathioprine

Chlorambucil

Cyclosporine A
Please select an answer
Feedback: The alkylating drugs chlorambucil and cyclophosphamide are associated with a dose-dependent risk of sterility
in both men and women. Sperm and ova banking should be discussed with patients requiring these medications.

Question 14 of 130
Which of the following parasites is associated with diffuse unilateral subacute neuroretinitis?

Taenia solium

Baylisascaris procyonis

Toxocara catis

Toxoplasma gondii
Please select an answer
Feedback: DUSN is an uncommon condition usually presenting with visual changes, photopsias, floaters, and progressing
to field loss with a 'unilateral wipeout' appearance to the fundus, similar to retinitis pigmentosum. The roundworm
Baylisascaris procyonis, which is carried by raccoons, has been implicated in this condition.
Question 15 of 130
Which form of juvenile idiopathic arthritis is most likely to be associated with anterior uveitis?

Pauciarticular

ANA-; RF+

Still's disease

Polyarticular
Please select an answer
Feedback: Between 25% and 40% of ANA+, pauciarticular onset patients with juvenile arthritis will develop uveitis. The
uveitis associated with this disorder tends to be asymptomatic, but very persistent, and is associated with very high rates of
complication.

Question 16 of 130
Which of the following is not a correct HLA association of uveitic diseases?

Birdshot chorioretinopathy = HLA A29

Tubulointerstitial nephritis and uveitis syndrome = HLA DRB1

Fuchs heterochromic cyclitis = HLA DR-4

Acute anterior uveitis = HLA B27


Please select an answer
Feedback: Birdshot choroiditis (HLA-A29), acute anterior uveitis (HLA-B27), and tubulointerstitial nephritis and uveitis
syndrome (TINU, HLA-DRB1*0102) all have extremely high relative risks associated with their specific HLA alleles. Fuchs
heterochromic cyclitis does not have a known HLA association.

Question 17 of 130
Initial treatment should not include immunomodulatory therapy in addition to corticosteroids for which of the following?

Wegener's granulomatosis

Sarcoidosis

Mucous membrane pemphigoid

Behcet's disease
Please select an answer
Feedback: Certain uveitic conditions including Wegener's granulomatosis, mucous membrane pemphigoid, Behcet's
disease, and foveal-threatening serpiginous choroiditis need to be treated from onset with immunomodulatory drugs.
Sarcoidosis frequently responds to corticosteroid therapy alone.
Question 18 of 130
Which of the following diseases is not associated with HLA-B27 positivity?

Psoriatic arthritis

Reactive arthritis

Ankylosing spondylitis

Rheumatoid arthritis
Please select an answer
Feedback: Ankylosing spondylitis, reactive arthritis, psoriatic arthritis, inflammatory bowel disease, and undifferentiated
spondyloarthropathy are all HLA-B27 associated diseases that may all be associated with anterior uveitis. Rheumatoid
arthritis does not have a strong Class I HLA association, and is more typically associated with scleritis or keratitis than
anterior uveitis.

Question 18 of 130
Which of the following diseases is not associated with HLA-B27 positivity?

Psoriatic arthritis

Reactive arthritis

Ankylosing spondylitis

Rheumatoid arthritis
Please select an answer
Feedback: Ankylosing spondylitis, reactive arthritis, psoriatic arthritis, inflammatory bowel disease, and undifferentiated
spondyloarthropathy are all HLA-B27 associated diseases that may all be associated with anterior uveitis. Rheumatoid
arthritis does not have a strong Class I HLA association, and is more typically associated with scleritis or keratitis than
anterior uveitis.

Question 19 of 130
What is the life stage of Toxoplasma gondii associated with reactivated disease?

Tachyzoite

Cystozoite

Oocyst

Bradyzoite
Please select an answer
Feedback: The form of T. gondii responsible for uveitic disease is the tachyzoite, an asexually dividing life stage of the
parasite. Infection is typically from ingestion of oocytes, or the sexually reproducing egg form. Bradyzoites are a cyst form of
the organism which can persist latent for many years. Cystozoite is an older term for bradyzoite.
Question 20 of 130

An 84-year-old woman with a history of bilateral pseudophakia presents with a 7-day history of sudden, decreased vision in
her right eye without associated pain, redness, or photophobia. Examination reveals best corrected visual acuity of counting
fingers at 2 feet OD and 20/25 OS. Examination of the right eye shows an afferent pupillary defect, inferior keratic
precipitates, and mild flare and cell. The vitreous in the right eye contains clumps of white, cellular debris. Posterior chamber
IOLs are in position with central posterior capsular openings OU. The left eye is normal. The patient was diagnosed 4 years
ago with tuberculosis, and was treated medically for it for 1 year. Which one of the following is least likely to be helpful in
establishing a diagnosis in this patient?

Fluorescein angiography

Magnetic resonance imaging (MRI) of the head and orbits

Vitreous biopsy for cytology and microbiology

Referral to an internist for evaluation of possible systemic malignancy, immunosuppression, or reactivation of


tuberculosis
Please select an answer
Feedback: This patient presents with panuveitis, including vitritis, retinal vasculitis, and retinitis, with involvement of the
optic nerve head. Given the patient's age, clinical presentation, ocular findings, and previous history of tuberculosis, an
infectious or neoplastic cause is most likely. Tuberculosis would be high in the differential diagnosis. Other organisms,
particularly fungal, should also be strongly considered. The patient is not known to be immunosuppressed, but a thorough
evaluation, including complete blood count with differential and HIV testing, should be performed. Evidence of extraocular
malignancy or active tuberculosis should also be evaluated. MRI may provide information about involvement of the optic
nerve or central nervous system. Vitrectomy for vitreous biopsy should be performed in this patient, and the specimen sent
for both microbiologic and cytologic examination. Although there is clinical evidence of retinal vascular inflammation,
fluorescein angiography is unlikely to provide any useful information in establishing a diagnosis.

Question 21 of 130
Which of the following drug side-effect relationships is incorrect?

Methotrexate -- hepatotoxicity

Infliximab -- exacerbation of multiple sclerosis

Cyclosporine A -- hypertension

Azathioprine -- sterility
Please select an answer
Feedback: Azathioprine is associated with bone marrow and hepatic toxicity. Alkylating agents are associated with sterility.
Cyclosporine A side effects include hirsutism, tremor, hypertension, and renal dysfunction. Infliximab is associated with
tuberculosis reactivation, worsening of multiple sclerosis, and possible increased risk of lymphoma. Methotrexate is
associated with hepatotoxicity and rarely pulmonary fibrosis.
Question 22 of 130
Which of the following disorders is not always bilateral?

Pars planitis

Vogt-Koyanagi-Harada disease

Birdshot chorioretinopathy

Sympathetic ophthalmia
Please select an answer
Feedback: Birdshot chorioretinopathy, VKH, and sympathetic ophthalmia are presumed autoimmune conditions that always
affect both eyes. While 90% of pars planitis cases are bilateral, a unilateral form of the disease has been described.

Question 23 of 130
What is the predominant cell type seen in the anterior chamber of a patient with phacolytic uveitis and glaucoma?

Neutrophil

T-lymphocyte

Macrophage

Eosinophil
Please select an answer
Feedback: Phacolytic glaucoma occurs when lens proteins leech from liquified cortex of a hypermature cataract into the
anterior chamber. Macrophages respond to this protein and engulf it, becoming 'lipid laden'. These cells clog the trabecular
meshwork, leading to glaucoma. Treatment requires acute corticosteroids, pressure lowering agents, and prompt removal of
the cataract.

Question 24 of 130
What would the differential diagnosis for isolated intermediate uveitis not include?

Reactive arthritis

Primary CNS lymphoma

Multiple sclerosis

Lyme disease
Please select an answer
Feedback: The differential diagnosis for isolated intermediate uveitis includes multiple sclerosis (found in 10-30% of patients
with intermediate uveitis), Lyme disease, sarcoidosis, syphilis, tuberculosis, intraocular lymphoma, and ocular ischemic
syndrome. Reactive arthritis is more typically associated with conjunctivitis and anterior uveitis.
Question 25 of 130
The 'white dot' lesions of which disease is least apparent on fluorescein angiography?

Birdshot chorioretinopathy

Serpiginous choroiditis

Punctate inner choroidopathy (PIC)

Acute posterior multifocal placoid pigment epitheliopathy (APMPPE)


Please select an answer
Feedback: APMPPE, PIC, and serpiginous choroiditis lesions generally show a 'early block, late stain' pattern on
fluorescein angiography. In contrast, birdshot choroiditis lesions tend to be angiographically silent in the active phases of
disease. They may be visualized better with indocyanin green angiography.

Question 26 of 130
Early findings in Vogt Koyanagi Harada (VKH) syndrome does not include which of the following?

Tinnitus

Granulomatous anterior segment inflammation

Vitiligo

Bilateral serous retinal detachments


Please select an answer
Feedback: The prodromal and acute phases of VKH typically include CNS findings such as meningismus and headache,
auditory findings including vertigo, tinnitus, and dysacoustia, and ocular findings including granulomatous inflammation and
serous retinal effusions. Skin findings such as vitiligo, alopecia, and poliosis tend to be late findings, as is the 'sunset glow'
fundus.

Question 27 of 130
Which of the following uveitic conditions is most likely to present with a white and asymptomatic eye?

Herpetic anterior uveitis

Acute retinal necrosis syndrome

Wegener's granulomatosis

Juvenile idiopathic arthritis-associated uveitis


Please select an answer
Feedback: Juvenile idiopathic arthritis-associated uveitis typically presents with a white eye and no symptoms. Patients
diagnosed with JIA, particular the pauciarticular, ANA+ variety, must undergo regular eye examinations. The other
conditions here would all typically present with anterior segment injection, pain, and photophobia.
Question 28 of 130
A purified protein derivative (PPD) test greater than ____ mm is considered positive in all individuals.

2 mm

15 mm

5 mm

10 mm
Please select an answer
Feedback: The PPD is read 48-72 hours after placement, and induration measured. A reading greater than 15 mm is
considered positive in all individuals. In individuals at risk, including health care workers or potential contacts with active
cases, 10 mm is considered positive. In individuals with AIDS or positive chest X-ray findings, 5 mm is considered positive.
A 2 mm PPD would be considered negative.

Question 29 of 130
What are Dalen-Fuchs nodules associated with?

Eales disease

Sympathetic ophthalmia

Ocular sarcoidosis

Behcet disease
Please select an answer
Feedback: Dalen-Fuchs nodules are small, yellow-white excrescences located between Bruchs' membrane and the retinal
pigment epithelium. They are associated with sympathetic ophthalmia and Vogt-Koyanagi-Harada syndrome.

Question 30 of 130
The fluocinolone acetonide implant releases therapeutic levels of corticosteroids to the vitreous cavity for approximately how
many days?

500

5000

1000

100
Please select an answer
Feedback: The fluocinolone acetonide implant (Retisert) releases therapeutic levels of drug for approximately 1000 days
(~3 years). Reactivation rates for intermediate and posterior uveitis are much lower in treated eyes than in untreated
controls. However, ~90% of phakic patients will develop cataract with this device and ~40-50% will require glaucoma
surgery.
Question 31 of 130
Which of the following tests is the most sensitive mode of work-up for a patient with the presentation associated with chronic
epistaxis and sinusitis in the attached images?

Serology

Scleral culture

Scleral biopsy

Magnetic resonance imaging (MRI) of orbits


Please select an answer
Feedback: Necrotizing scleritis is a destructive type of scleritis that is associated with systemic vasculitic diseases, such as
Wegener's granulomatosis. Serological assay for c-ANCA, or anti-neutrophil cytoplasmic antibody, is a highly sensitive test
to confirm the diagnosis of WG. If positive, subsequent testing for myeloperoxidase and/or proteinase-3 is generally
performed. Serology is more sensitive than orbital imaging and scleral biopsy.

Question 32 of 130
Which of the following is a preferred method of treatment in a patient with primary central nervous system/intraocular
lymphoma?

Systemic corticosteroid therapy

Systemic chemotherapy

Intrathecal methotrexate and local radiopathy

Local radiopathy
Please select an answer
Feedback: Intraocular lymphoma represents a form of primary central nervous system lymphoma. Treatment must be
directed at both ocular lesion and potential CNS lesions. Ocular modalities include radiotherapy and intravitreal
methotrexate. Systemic treatments include chemotherapy and intrathecal methotrexate

Question 33 of 130
Which of the following patients is most likely to have scleritis?

A 40-year-old male with red eye and throbbing eye pain with weight loss and non-healing skin lesions

A 30-year-old woman with red eye and blurred vision

A 56-year-old woman with foreign body sensation and red eye

A 73-year-old male with red eye and decreased vision with ipsilateral headaches and jaw pain
Please select an answer
Feedback: Scleritis is often associated with systemic vasculities, including polyarteritis nodosum, Wegener's
granulomatosis, Churg-Strauss syndrome, and systemic lupus erythematosis. Weight loss and skin lesions would be
consistent with a systemic vasculitis.
Question 34 of 130
Which of the following statements best describes the epidemiology of scleritis associated with collagen vascular disease?

Men are more commonly affected than women.

Bilateral involvement is seen only rarely.

About half of the patients have an underlying systemic medical condition.

It is most common in third decade.


Please select an answer
Feedback: About 40 to 50 percent of patients with scleritis will have an underlying collagen vascular disease or infectious
etiology, most commonly rheumatoid arthritis.

Question 35 of 130
Most patients presenting with acute anterior nongranulomatous anterior uveitis will require treatment with which of the
following?

Topical corticosteroids and cycloplegic agents only

Periocular corticosteroids injections

Oral corticosteroids

Immunosuppressive agents orally


Please select an answer
Feedback: Most acute anterior uveitis responds well to topical therapy. In recalitrant cases, addition of oral corticosteroids
can hasten resolution, and have the advantage of being rapidly tapered if the patient is intolerant. Corticosteroid
monotherapy is relatively contraindicated in herpes virus-associated uveitis. Oral antiviral medication should be utilized in
conjunction with local and systemic corticosteroids in this case.

Question 36 of 130
Which of the following patients is most likely to have primary central nervous system/intraocular lymphoma?

A 59-year-old male with hemorrhagic retinitis and retinal vasculitis

A 29-year-old female with pars plana exudates and retinal vasculitis

A 40-year-old male with cotton wool spots and hard exudates

A 65-year-old female with dense vitritis, subretinal infiltrates, and mental confusion
Please select an answer
Feedback: Subretinal infiltrates and mental confusion are typical for primary central nervous system/intraocular lymphoma.
The patient's age is also compatible.
Question 37 of 130
Which one of the following tests is most important for patients with either intermediate or posterior uveitis, but without
keratitis or scleritis?

Treponema-specific syphilis serologies

Serum rheumatoid factor

Serum antinuclear antibodies

HLA-B27
Please select an answer
Feedback: It is always important to rule out syphilis, a treatable disease using anti-treponemal specific testing. HLA-B27 is
rarely associated with posterior uveitis without a significant anterior uveitis, but is most often associated with recurrent
unilateral acute anterior uveitis. In any case, a positive HLA-B27 test does not make a specific diagnosis, as almost 10% of
the Caucasian population is HLA-B27 positive, so it would be little help here. Similarly, anti-nuclear antibodies can be found
in normal individuals. Neither rheumatoid arthritis (RA) nor systemic lupus erythematosis (SLE) is commonly associated with
uveitis without scleritis or retinal vasculitis. If either are suspected, a good history and physical exam are more important
than these serologies.

Question 38 of 130
Which of the following statements best describes the demographics of acute posterior multifocal placoid epitheliopathy?

The male to female ratio is greater than 1.

Young, healthy adults are frequently affected.

It has a seasonal predilection.

It is a disease that affects caucasians only.


Please select an answer
Feedback: AMPPE tends to be a disease of young, healthy individuals. It may have a viral prodrome. Unlike punctate inner
choroiditis, it does not have a female prediliction

Question 39 of 130
Which of the following is a complication of punctate inner choroidopathy?

Choroidal neovascularization

Optic atrophy

Cystoid macular edema

Retinal detachment
Please select an answer
Feedback: Up to 40% of patients with punctate inner choroidopathy (PIC) will develop choroidal neovascularization (CNV).
Inflammation in PIC is limited to the inner choroid, the vitreous is generally without inflammation, and other complications are
very rare with this condition.
Question 40 of 130
Which of the following clinical features are more common in scleritis than episcleritis?

It blanches in response to topical phenylephrin, while episcleritis does not

It is potentially progressively destructive and sight threatening if left untreated

It is simple or nodular, and is not associated with systemic collagen-vascular disease

Minimal tenderness compared with episcleritis


Please select an answer
Feedback: Both episcleritis and scleritis are associated with collagen-vascular diseases, among other etiologies. Scleritis is
generally progressive if left untreated. It typically features deep, violaceous redness and tenderness to palpation. Scleritis
may be nodular or diffuse. Episcleritis, but not scleritis, typically blanches in response to topical phenylephrin.

Question 41 of 130
Mycophenolate mofetil is not appropriate for treatment of which of the following conditions?

Fuchs’ iridocyclitis syndrome

Birdshot retinochoroidopathy

Sympathetic ophthalmia

Rheumatoid arthritis-associated scleritis


Please select an answer
Feedback: Anti-inflammatory therapy is not generally necessary for Fuchs’ iridocyclitis syndrome. Mycophenolate mofetil is
commonly used as part of long-term, steroid-sparing, immunosuppressive therapy for the other conditions listed. It should be
noted that this is an off-label use of this agent.

Question 42 of 130
Which of the following statements best describes the typical fundus findings of serpiginous choroiditis?

Old, scarred lesions may be present in the newly-diagnosed eye.

Intense vitritis is common.

Multiple, new, isolated lesions occur with recurrences.

Vasculitis is a prominent feature.


Please select an answer
Feedback: Serpiginous choroiditis is a typically recurrent, bilateral disease that can lead to significant vision loss if
untreated. Inflammation of the choriocapillaris leads to loss, which is typically by contiguous spread, although new foci can
appear. Minimal, if any, vitritis is present, and retinal vasculitis is not seen. Treatment is with immunomodulatory drugs.
Question 43 of 130
A 39-year-old man with AIDS and cytomegalovirus (CMV) retinitis in 1 eye is treated with valganciclovir, but declines highly
active antiretroviral therapy. After 2 months, the retinitis is inactive. Nine months after beginning therapy, the retinitis is
active, and the patient's CD4+ count is 15 cells/µL. He undergoes placement of a ganciclovir implant. Two months later, the
retinitis remains active. What would the most appropriate approach at this time be?

Begin weekly intravitreous ganciclovir injections

Allow more time for the ganciclovir implant to take effect

Place another ganciclovir implant

Begin intravenous foscarnet


Please select an answer
Feedback: The patient has likely developed ganciclovir-resistant CMV retinitis. Placing another ganciclovir implant, or giving
intravitreous ganciclovir injections, is unlikely to be effective. Ganciclovir-resistant CMV retinitis may respond to intravenous
foscarnet.

Question 44 of 130
A 5-year-old patient with juvenile chronic arthritis and chronic anterior uveitis was started on methotrexate and achieved
control of the ocular inflammation for the first time in 6 months of therapy. However, the child now complains of abdominal
pains and refuses to take the medicine. Which approach would most likely succeed in this setting?

Have the parent administer the drug by subcutaneous injection

Administer the methotrexate pills at bedtime, so the child can sleep through the period of discomfort

Mix the methotrexate pills with a favorite food

Switch to an alternative immunosuppressive agent


Please select an answer
Feedback: Use of corticosteroids carries significant morbidity for pediatric patients, including growth retardation, cushingoid
changes, and endocrine abnormalities. Methotrexate has long been used by rheumatologists as a steroid sparing agent for
juvenile arthritis, and has a good safety profile in this population. Subcutaneous injection of methotrexate may alleviate
gastrointestinal side effects of the drug. It also may improve compliance and offer slightly improved efficacy as first-pass
hepatic metabolism is bypassed. The drug is given weekly, usually at a dose of 7.5 mg - 20 mg, titrated to effect. Many
rheumatologists also favor use of folic acid (600 - 800 mcg daily), which may decrease side effects of methotrexate.

Question 45 of 130
Which of the following is the most appropriate treatment for a 30-year-old man with sinus inflammation, bilateral scleritis,
mild anterior uveitis, and a positive anti-neutrophilic cytoplasmic antibodies (c-ANCA) test?

Oral non-steroidal, anti-inflammatory agents such as indomethacin at 50 mg tid

Topical non-steroidal anti-inflammatory agents such as ketralac

Injection of 40 mg of periocular triamcinolone actetate

Oral cyclophosphamide at 2 mg/kg/day


Please select an answer
Feedback: Several ocular inflammatory conditions mandate initial treatment with strong immunomodulation. These
conditions include Wegeners granulomatosis, serpiginous choroiditis, Behcet disease, and mucous membrane pemphigoid.
Treatment options for these conditions include alkylator therapy or transplant-level immunomodulation with cyclosporine,
azathioprine, and prednisone.

Question 46 of 130
Which of the following is a recommended treatment for serpiginous choroiditis involving the macula?

Oral corticosteroids

Topical corticosteroids

Acyclovir

Alkylating agents
Please select an answer
Feedback: Vision threatening macular serpiginous choroiditis requires prompt institution of strong immunomodulatory
therapy. Success has been reported for alkylating agents and for use of transplant-dose therapy with cyclosporine A,
azathioprine, and prednisone. Corticosteroids alone are often insufficient to treat this condition. While there has been some
discussion suggesting a herpetic genesis for serpiginous choroiditis, definitive evidence has not been demonstrated, and
acyclovir should be used adjunctively if at all. Topical corticosteroids have no effect on this condition.

Question 47 of 130
A 25-year-old, Brazilian man presents with a history of decreased vision in his left eye for 1 week. Visual acuity is 20/70, and
moderate vitreitis is present. On dilated examination, a pigmented scar in the posterior pole with adjacent yellow white
chorioretinitis is present. What is the most appropriate treatment?

Pyrimethamine, sulfadiazine, and folinic acid

Amphotericin B

IV acyclovir

Oral corticosteroids
Please select an answer
Feedback: This patient likely has ocular toxoplasmosis. While most ocular toxoplasmosis is thought to be reactivation, and
will be accompanied by a chorioretinal scar, recent work has suggested that de novo infection can also be associated with
uveitis, particularly in endemic areas, such as southern Brazil. Appropriate treatments for ocular toxoplasmosis include
combination pyrimethamine, sulfadiazene, folinic acid; trimethoprim-sulfamethoxazole; azithromycin; and clindamycin (or
combinations thereof). Amphotericin B is an antifungal agent; IV acyclovir is more appropriate for acute retinal necrosis due
to herpes virus; and oral corticosteroids without antibiotic use are contraindicated with active ocular toxoplasmosis.

Question 48 of 130
The daily dosage of cyclosporine should be adjusted based on which of the following laboratory tests?

Hematocrit

Glucose

Creatinine

Platelet count
Please select an answer
Feedback: Cyclosporine A is potentially nephrotoxic. Blood pressure and renal function should be monitored every 4-6
weeks in all patients on this medication. Cyclosporine does not have direct effects on hematocrit, platelet count, or blood
glucose.

Question 49 of 130
A 39-year-old man with AIDS and cytomegalovirus (CMV) retinitis in 1 eye undergoes successful placement of an intravitreal
ganciclovir implant without systemic anti-CMV therapy. He declines highly active antiretroviral therapy. After 1 month, the
retinitis is inactive. Nine months after surgery, the retinitis is active, and the patient's CD4+ count is 20 cells/µL. He feels well
in general, and the opposite eye shows no retinitis. What is the most appropriate approach at this time?

Make no changes in therapy

Begin oral acyclovir

Begin intravenous foscarnet

Place another ganciclovir implant


Please select an answer
Feedback: This patient demonstrates recurrence of retinitis following exhaustion of the drug in the ganciclovir implant, which
usually provides active drug delivery for 7-8 months. Because the patient’s CD4+ count remains low, additional anti-CMV
therapy is indicated. The first implant can be left in place and another implant placed in a different quadrant. Intravenous
foscarnet requires placement of a permanent indwelling catheter for daily infusions and is a less preferable approach.
Acyclovir is not effective for the treatment of CMV retinitis. It is worth noting that mortality rates are higher for patients with
CMV retinitis who are not placed on systemic anti-CMV therapy; for this reason and for fellow eye prophylaxis, it would also
be desirable to place the patient on oral valganciclovir.

Question 50 of 130
In which of the disorders below is indocyanine green (ICG) angiography most useful?

Pars planitis

Primary central nervous system/intraocular lymphoma

Posterior scleritis

Serpiginous choroiditis
Please select an answer
Feedback: ICG angiography is occasionally helpful in making the diagnosis of an individual with posterior uveitis. It can be
used, for example, in distinguishing macular serpiginous choroiditis from persistent placoid maculopathy. The former will
show resolution of the macular hypofluorescence, while the latter will show persistent hypofluorescence. That said, the utility
of ICG angiography is generally less than fluorescein angiography, as it does not show aspects of retinal circulation or
disease.
Question 51 of 130
What infectious agent has been associated with Fuchs heterochromic iridocyclitis?

Syphilis

Rubella

Herpes simplex

Herpes zoster
Please select an answer
Feedback: Both epidemiologic and polymerase chain reaction studies suggest rubella is associated with Fuchs
heterochromic iridocyclitis. This is problematic, because we do not have good agents to treat rubella virus infections.

Question 52 of 130
A 28-year-old Hispanic woman presents with three days of headache and loss of vision in both eyes. She was admitted for
suspicion of meningitis, although she has no fever. There was a mild lymphocytic response noted in her cerebrospinal fluid;
stains for microbes were negative, and cultures are negative to date. On examination she has a moderate bilateral anterior
uveitis and vitritis with serous retinal detachments. There is no retinal necrosis. What is the most appropriate therapy?

Intravitreal acyclovir

Systemic corticosteroids

Cyclophospahmide

Await viral cultures of the cerebrospinal fluid before starting treatment


Please select an answer
Feedback: This is a typical presentation of Vogt-Koyanagi-Harada disease, for which the treatment is high-dose systemic
corticosteroids.

Question 53 of 130
A 14-year-old girl complains of a sudden onset of bilateral eye pain and photophobia. On ocular examination she has
bilateral anterior uveitis. She has had a fever for two weeks, modest flank and abdominal pain, and weight loss of 10
pounds. Laboratory testing reveals an elevated erythrocyte sedimentation rate at 80 mm/hour, mild elevation of the serum
creatinine, and mild proteinuria. What is the most likely diagnosis?

Lyme disease

Toxocara uveitis

Tubulointerstitial nephritis and uveitis

Toxoplasmosis
Please select an answer
Feedback: This case illustrates many of the common features of this rare disease. In fact, there is evidence that the
diagnosis is often missed, and that Tubulointersitital nephritis and uveitis may not be as rare as we have thought in the past.
Toxacara uveitis is not bilateral and not usually associated with systemic disease. Lyme disease can be considered, but the
sudden onset of illness without sings (such as a typical rash), or history consistent with lyme disease (or the more typical
intermediate uveitis), makes it less likely. Toxoplasmosis does not cause a bilateral anterior uveitis, and systemic
toxoplasmosis would be very uncommon in this age group.

Question 54 of 130
A 35-year-old man has decreased vision, pain, redness, and photophobia in the left eye of three days duration. He is
otherwise healthy, without known systemic disease. On examination the right eye is without inflammation. Examination of
the left eye reveals 3+ anterior chamber cells and 2-3+ vitreous cells, and haze with 5 clock hours of retinitis anterior to the
equator. He is HIV negative. What is the most likely diagnosis?

Viral acute retinal necrosis

Toxoplasmosis

Toxocara

Sarcoidosis
Please select an answer
Feedback: A rapid onset of symptomatic retinitis anterior to the equator in a non-immunosuppressed patient best fits the
description of viral acute retinal necrosis. Toxocara would be an unusual cause or retinal necrosis, especially in this age
group. Sarcoidosis is not the differential diagnosis, but is less likely with this picture. A bilateral granulomatous pan uveitis,
not quite as rapid in its course, would be much more typical, although retinal necrosis is possible. Toxoplasmosis is unlikely
to result in such a large area of retinal necrosis unless there is immunosuppression, including corticosteroid injection.

Question 55 of 130
What is the mechanism of action of infliximab?

Corticosteroid

Alkylating agent

Antimetabolite

Anti-tumor necrosis factor antibody


Please select an answer
Feedback: Infliximab is a murine antibody to tumor necrosis factor (TNF) that has been effective in the treatment of uveitis.
It may be used with antimetobolites, both for increased effectiveness and to try to prevent anti-infliximab antibodies,
although this is not always done. Other TNF agents include adalimumab (a humanized anti-TNF antibody) and etanercept (a
pseudoreceptor). There is less experience with adalimummab, but it may not be as effective as infliximab in treating uveitis;
etanercept certainly is not.

Question 56 of 130
What infectious agent(s) has been associated with corneal endothelial and subendothelial changes, leading to corneal
edema in an eye with recurrent unilateral anterior uveitis, in a patient who is not immunosuppressed?

Toxoplasmosis

Herpes viruses and cytomegalovirus

Bartonella

Rubella
Please select an answer
Feedback: Cytomegallovirus anterior uveitis has been shown by polymerase chain reaction techniques to be associated
with anterior uveitis, often with endothelial changes and corneal edema that can recur in corneal grafts, in non-
immunosuppressed patients. Treatment is problematic, because valganciclovir is much more toxic than acyclovir or other
agents that are effective for herpes zoster or herpes simplex. Rubella has been associated with Fuchs heterochromic
iridocyclitis. Bartonella and toxoplasmosis ocular disease rarely, if ever, would present with an isolated unilateral anterior
uveitis leading to corneal edema.

Question 57 of 130
A patient is being evaluated by a rheumatologist for the possibility of sarcoidosis. While the patient has no ocular complaints
and has a normal eye exam, the rheumatologist requests that a conjunctival biopsy be performed to help confirm the
diagnosis of sarcoidosis. Which of the following is the most appropriate response to the rheumatologist?

"Conjunctival biopsy is contraindicated in all patients with sarcoidosis."

"We should rely on laboratory testing, such as angiotensin converting enzyme or gallium scan, instead of biopsy."

"There are no alternatives to the conjunctiva in making a tissue diagnosis of sarcoidosis."

"Blind conjunctival biopsies without a visible lesion will have a low diagnostic yield."
Please select an answer
Feedback: Sarcoidosis is a tissue diagnosis, and efforts should be made to identify involved tissues for biopsy. If
conjunctival granulomas are observed, they may be biopsied with high diagnostic yield. In the absence of visible conjunctival
lesions, however, biopsy yields are less than 10%. Higher yield may be obtained by chest CT scanning followed by
mediastinoscopy or bronchoscopy. However, such invasive diagnostic tests should be considered only when making the
sarcoidosis diagnosis has systemic treatment implications for the patient.

Question 58 of 130
Symptoms of serpiginous choroidopathy include which of the following?

Scotoma

Pain

Redness

Photophobia
Please select an answer
Feedback: Serpiginous choroiditis causes acute inflammation of the choroid and choriorcapillaris, with subsequent loss of
retinal pigment epithelium and photoreceptors. The resulting retinal dysfunction manifests as scotoma. Pain, photophobia,
and redness are typically associated with anterior uveitis.
Question 59 of 130
A 64-year-old Caucasian woman presents with a history of floaters and mildly blurred vision for two weeks in her left eye. On
examination of the right eye there is no inflammation. The left eye has mild anterior uveitis, moderate vitritis, and a localized
area of retinitis of about one half disc diameter in size nasal to the optic nerve head, but posterior to the equator of the
globe. The optic nerve head is not involved. She is otherwise healthy with no known systemic disease and is on no
medications. What is the most likely diagnosis?

Behcets disease

Acute retinal necrosis

Toxoplasmosis

Toxocara
Please select an answer
Feedback: Toxoplasmosis is the most common cause of unilateral retinitis in this population. Toxocara would be unlikely in
this age group. Acute retinal necrosis is most often posterior to the equator, and after two weeks would have likely enlarged
ion a circumfrential fashion. Behcets disease is unlikely in a middle aged Caucasian woman without other findings and is
certainly less likely than toxoplasmosis.

Question 60 of 130
A 34-year-old man has a localized area of scleral thinning 5 mm nasal to the limbus. The surrounding episcleral and
conjunctival tissue is not injected. The patient had eye surgery four years before presentation. He is healthy without any
systemic inflammatory or infectious disease. What is the most likely surgery he had that could result in this finding?

Pterygium excision with mitomycin C or Beta-irradiation

Trabeculectomy

Laser retinopexy

LASIK
Please select an answer
Feedback: Necrotizing scleritis certainly raises concerns about systemic disease, but one must consider other causes
(foreign bodies, infection, previous mitomycin or radiation). The nasal position would make a trabeculectomy with mitomycin
unlikely, but is a typical location for pterygia.

Question 61 of 130
A patient with a bilateral anterior and intermediate uveitis is suspected of having sarcoidosis. There are no conjunctival or lid
granulomata. A chest x-ray shows no abnormalities, and serum angiotensin converting enzyme is normal. Which one of the
following is the most appropriate examination for pursuing the diagnosis of sarcoidosis?

High-resolution CT scan of the chest

HLA-B27 status

Biopsy of the conjunctiva

Repeat serum angiotensin converting enzyme to rule out laboratory error


Please select an answer
Feedback: The most likely diagnosis is sarcoidosos. Both chest x-ray and angiotensin-converting enzymes are often used
to screen for sarcoidosis, but they are neither very sensitive nor specific. Biopsy of the conjunctiva, unless there are
suspicious lesions, has a very low yield.

Question 62 of 130
Which one of the following is most important in diagnosing Behcet's disease?

HLA-B51 status

The presence of a hypopyon on slit lamp examination

Findings of ischemia on indocyanine green angiography

A systemic history and physical examination


Please select an answer
Feedback: While the diagnositc criteria for Behcet's disease include uveitis consistent with the diagnosis, that is not
sufficient to make the diagnosis. Although a hypopyon can be associated with Behcet's disease, it is neither sensitive nor
specific for the diagnosis; i.e., most eyes with hypopyon do not have Behcet's disease, and many patients with Behcet's
disease do not have hypopyon. Similarly, ischemia on the indocyanine green test is not specific for Behcet's. Much more
sensitive (although not necessarily specific) are the major criteria: presence of oral or genital ulcers, or skin lesions, along
with ocular inflammation. The minor criteria are arthritis, ulceration of the bowel, epididymitis, vasculitis, and
neuropsychiatric symptoms. HLA-B51 has a somewhat low relative risk for Behcet's disease and is not part of the diagnostic
criteria; while it is a reasonable test to obtain, it is not sufficient or necessary for the diagnosis.

Question 63 of 130
A 47-year-old woman with birdshot chorioretinopathy has 20/40 Snellen visual acuity. She has received no treatment for this
disease. What is the most likely case of decreased Snellen visual acuity?

Cataract

Cystoid macular edema

Retinal vasculitis

Glaucoma
Please select an answer
Feedback: The most common cause of decreased visual acuity in birdshot chorioretinopathy is cystoid macular edema.
Glaucoma or cataract formation is unusual in birdshot chorioretinopathy except secondary to corticosteroids, because there
is little anterior uveitis. Retinal vasculitis is common in birdshot chorioretinopathy, but is not commonly associated in itself
with decreased visual acuity.
Question 64 of 130
A 23-year-old man presents complainig of floaters in his left eye for 2 weeks. The visual acuity is 20/20 in both eyes. There
is a mild (1+) anterior chamber and vitreous reaction, with a small focus of retinitis (less than 1/2 disc diameter) adjacent to a
chorio-retinal scar, anterior to the vascular arcades in the left eye. He is HIV negative. He is allergic to sulfa medication.
What is the most appropriate next step?

Intravitreal clindamycin and dexamethasone

Oral clindamycin, and after 2 days of therapy, start oral prednisone

Hospitalization for intravenous acyclovir

Observe closely without treatment


Please select an answer
Feedback: Not every reactivation of toxoplasmosis retinochoroiditis has to be treated; it is difficult for immunocompetent
patients even to show that treatment has much effect. Certainly, lesions that are sight-threatening (ie, in the macula or by
the optic nerve head) are most often treated, as well as lesions that are large and associated with significant vitritis (if for no
other reason than to be able to give corticosteroids). If he were HIV positive, particualrly with a low CD4 count, treatment
and evaluation for systemic or central nervous system toxoplasmosis would be indicated. This could be early viral acute
retinal necrosis and close follow-up is indicated, but the adjacent chorioretinal scar makes that highly unlikely.

Question 65 of 130
Retinal vasculitis affecting only the veins is most likely to be associated with which of the following?

Behcet disease

Systemic lupus erythematosis

Ocular sarcoidosis

Polyarteritis nodosa
Please select an answer
Feedback: A pure retinal periphlebitis is most likely to be associated with ocular sarcoidosis. The form of this inflammation
has been called 'candlewax drippings' or Taches de Bougie. Polyarteritis nodosa is typically associated with arteritis in the
eye. Systemic lupus erythematosis and Behcets disease may present with mixed arterial and venous inflammation.

Question 66 of 130
Which of the following approaches is most appropriate for cataract surgery in a patient with Fuch's heterochromic iridocyclitis
without glaucoma?

Standard phacoemulsification with a posterior chamber intraocular lens implant (IOL)

Start acyclovir, because Fuch's heterochromic iridocyclitis has been associated with rubella virus

Start methotrexate with a goal of no more than rare cell before surgery

Perform a lensectomy and vitrectomy, and leave the eye aphakic to prevent ciliary body membranes and hypotony
Please select an answer
Feedback: Eyes with Fuch's heterochromic iridocyclitis tend to do well with standard cataract surgery techniques and
intraocular lens (IOL) implant placement. Some uveitis specialists will treat briefly before surgery with topical and even oral
corticosteroids, but systemic immunomodulatory therapy such as one with methotrexate is not usually pursued as one would
pursue for chronic anterior uveitis associated with such diseases as sarcoidosis, VKH disease, or juvenile rheumatoid
arthritis. FHI has been associated with Rubella virus, but acyclovir is not an effective treatment.

Question 67 of 130
A 27-year-old man presents with a 2-day history of redness, pain, and photophobia in the left eye. The right eye is normal.
His Snellen visual acuity is 20/20 in the right eye and 20/25 in the left. In the left, there is a mild ciliary flush and 3+ cells in
the anterior chamber with no keratic precipitates or posterior synechiae. There is no posterior segment inflammation in either
eye. What is the most appropriate treatment?

Topical prednisolone acetate 1% 4 times/day, and posterior subtenon injection of 40 mg triamcinolone acetonide

Topical prednisolone acetate 1%, 4 times/day

Hourly prednisolone acetate 1%

Topical prednisolone acetate 1%, 4 times/day and oral prednisone 40 mg with a rapid taper
Please select an answer
Feedback: Although this patient may do quite well on a lower dose of topical corticosteroids, it is too early to tell, and so it is
better to treat aggressively with topical steroids as if a hypopyon and fibrin may develop. On the other hand, the patient most
likely will not need oral or injected corticosteroids, so there is no reason at this point to increase the risk of treatment by
using those modalities. He should be closely followed and the treatment tailored according to how his eye responds.

Question 68 of 130
Which one of the following tests is most likely to give a definitive diagnosis in a immunocompetent patient with a rapidly
progressing peripheral retinal necrosis in one eye?

Herpes serologies

Toxoplasma serologies

Vitreous for polymerase chain reaction testing for herpes viruses

Complete blood cell count with differential


Please select an answer
Feedback: Most acute retinal necrosis in immunocompromised individuals will be from herpes viruses, either simplex or
zoster. Putative auto-immune retinitis (such as Behcets disease), neoplasia (such as lymphomas), and other infections
(such as cytomegalovirus or toxoplasmosis), can rarely give a similar picture. Toxoplamsosis titers are reasonable, but not
as likely to give a definitive diagnosis; if negative they are reassuring (although there can be false negatives), but if positive,
they do not usually give a definitive diagnosis, because they mean only that the patient has had toxoplasmosis, but not that
it is causing ocular disease. This is also a very atypical presentation for toxoplasmosis in an immunocompetent patient
(although in AIDS patients toxoplasmosis can mimic CMV retinitis). A complete blood count is reasonable to obtain, because
it could reflect a systemic leukemia or lymphoma, but that is again much less common, and if intraocular, may not be
systemic. Herpes serologies are almost worthless, because the vast majority of the population has positive serologies, and
herpes serologies will not help determine the cause of the retinitis.
Question 69 of 130
A 6-year-old child presents with intermediate uveitis and a peripheral retinal granuloma in one eye. What is the most
appropriate diagnostic test?

Serum herpes simplex titers

Serum toxocara titers

A complete blood count (CBC) with differential

HLA-B27 status
Please select an answer
Feedback: The age of the patient and presentation are most consistent with toxocara. A complete blood count could show
abnormalities in various forms of uveitis, and possibly eosinophilia in toxocara, but other than in leukemia or systemic
lymphoma, it is not likely to lead to a specific diagnosis. This presentation is not typical of HLA-B27 disease and is postiive in
5-10% of the normal population, so even if the child is HLA-B27 positive, it would not be helpful. Similarly, herpes titers are
postivie in the majority of the population and a granuloma is not typical of herpetic disease.

Question 70 of 130
A patient with Vogt-Koyanagi-Harada disease was treated with high-dose oral prednisone and had resolution of her serous
retinal detachments. The dose has been gradually decreased, and 4 months into therapy, she is on prednisone 20 mg daily.
She is on topical prednislolone acetate 1% QID and her visual acuity is 20/30 both eyes. There are 2+ cells in the anterior
chamber of both eyes with no vitritis, cystoid macular edema, or subretinal fluid. What is the most appropriate next step?

Place a flucinolone implant

Posterior subtenon injections of triamcinolone acetonide

Increase the oral prednisone to 40 mg/day

Start a corticosteroid-sparing immunomodulatory agent


Please select an answer
Feedback: The goal generally is to have a patient on 10 mg prednisone per day or less by 6 months; if not, a corticosteroid-
sparing immunomodulatory agent is needed to allow further weaning of the prednisone. This is considered the standard of
care. Periocular injections do not tend to be as effective in the chronic anterior uveitis associated with VKH disease, but
could be considered. The flucinolone implant is not approved for anterior chamber inflammation.
Question 71 of 130
A 40-year-old man was evaluated for blurred vision in both eyes of 2 months duration. He had a previous episode of
sustained ventricular tachycardia, for which he received an implantable cardioverter defibrillator. The patient's cardiac
rhythm was controlled with the defibrillator and oral amiodarone. Six weeks prior to evaluation, the patient developed
amiodarone toxicity, and was hospitalized with multi-organ failure requiring intubation and a week in the intensive care unit.
On examination, his visual acuity was 20/100 OD, counting fingers OS, and 2+ cell and flare were present bilaterally in the
anterior chamber. Dilated funduscopic examination of the right eye revealed 2+ vitritis with a yellowish macular lesion and
epiretinal gliosis (part A of the figure). The left fundus showed a white inflammatory macular lesion with tractional macular
detachment with moderate overlying vitreous inflammation (part B). Which of the following is not a useful approach for the
treatment of this patient?
A toxoplasmosis titer

A vitreous tap or diagnostic vitrectomy

Blood cultures

An empirical trial of sub-Tenon's corticosteroids


Please select an answer
Feedback: Panuveitis can have a variety of causes, both infectious and noninfectious. Before starting empirical therapy with
corticosteroids, however, one needs to ensure that an infectious process is not present. With this patient's history of recent
hospitalization and indwelling catheters, an acquired infection such as candidiasis must be considered. Acquiring vitreous
fluid for microbiologic evaluation would be useful, as would obtaining blood cultures. This clinical picture might be consistent
with toxoplasmosis, particularly in an immunocompromised patient, and although a positive toxoplasmosis titer would not
confirm the diagnosis, a negative titer could rule it out. This patient underwent a vitreous tap, which failed to reveal fungal
elements. However, blood cultures revealed evidence of candidemia from an infected cardiac pacer wire. This patient was
treated with intravenous and intravitreal amphotericin, and the infected cardiac wire was removed. Candida endophthalmitis
in the immunocompetent host is usually attributable to systemic fungemia and must be diligently searched for in similar
clinical settings.
Question 72 of 130
A 26-year-old myopic man presents with a 5-day history of photopsias, small scotomas, and blurred vision in both eyes. He
is recovering from a recent flu-like illness. Examination reveals best-corrected visual acuity of 20/50 OD and 20/40 OS. Slit-
lamp examination shows mild flare and cell in both anterior chambers, and mild vitreous cell in both eyes. The fundus
findings are similar in both eyes; the right fundus is shown in the figure. Which of the following diagnoses is most likely in
this patient?

Acute posterior multifocal placoid pigment epitheliopathy

Multifocal choroiditis

Birdshot retinochoroidopathy

Presumed ocular histoplasmosis syndrome


Please select an answer
Feedback: The clinical picture in this patient represents an inflammatory process of the choroid and retina. Inflammatory
retinal and choroidal diseases are classified based on ophthalmoscopic findings and clinical course of the disease, with
overlap between many of the diagnoses. Of the choices given, multifocal choroiditis best fits the clinical history and
appearance in this patient. The patient's young age is consistent with any of the listed diagnoses except birdshot
retinochoroidopathy, which is more common in patients between ages 40 and 60. Bilateral ocular involvement is seen in all
of the diagnoses. A preceding viral illness is a frequent history given by patients with either multifocal choroiditis or acute
posterior multifocal placoid pigment epitheliopathy. Vitritis is a finding in multifocal choroiditis, birdshot retinochoroidopathy,
and acute posterior multifocal placoid pigment epitheliopathy. Acute posterior multifocal placoid pigment epitheliopathy
causes multiple yellow-white, flat, round or irregular lesions at the level of the pigment epithelium and choroid, typically
larger than the lesions present in this patient. Presumed ocular histoplasmosis syndrome commonly produces peripapillary
scarring (not present in this patient) in addition to typical punched-out, peripheral chorioretinal scars and lack of vitreous
cells.
Question 73 of 130
Which of the following statements is not accurate about Propionibacterium acnes endophthalmitis?
Inflammation associated with Propionibacterium endophthalmibs typically increases with topical corticosteroid use.

Propionibacterium may cause a recurrent granulomatous iridocyclitis that may not develop until months after cataract
surgery.

Propionibacterium acnes is frequently found in the normal flora of the conjunctiva.

Nd:YAG laser capsulotomy may cause a flare-up of the intraocular inflammation in Propionibacterium endophthalmitis.
Please select an answer
Feedback: Propionibacterium acnes is a Gram-positive, anaerobic, pleomorphic bacillus that is commonly present in the
normal conjunctival flora. It has been recognized as a cause of chronic, recurrent, often granulomatous inflammation
following cataract extraction. The inflammation typically associated with Propionibacterium acnes may not develop for weeks
to months after uncomplicated cataract surgery. The organism may be sequestered in the capsular bag and a characteristic
capsular plaque may be visible. Flare-up of intraocular inflammation has been reported following Nd:YAG laser
capsulotomy, which may expose or release more organisms. Inflammation from Propionibacterium acnes typically shows a
transient response to corticosteroids, but intraocular administration of antibiotic with or without vitrectomy, capsulectomy,
and/or IOL removal or exchange are the preferred treatments for Propionibacterium acnes endophthalmitis.
Question 74 of 130
What is the most common cause of decreased visual acuity from the inflammation in idiopathic intermediate uveitis?

Retinal detachment

Glaucoma

Vitreous hemorrhage

Cystoid macular edema


Please select an answer
Feedback: Multiple studies confirm cystoid macular edema is the most common cause of decreased vision in patients with
intermediate uveitis. Glaucoma is often due to corticosteroid treatment, though a small number of patients can have
sufficient anterior chamber inflammation to form enough synechiae to result in a uveitic glaucoma by that mechanism.
Vitreous hemorrhage and retinal detachment can occur, presumably secondary to vitreous traction from the virtitis or from
organized vitreous secondary to previous inflammation.

Question 75 of 130
A 28-year-old man with acquired immunodeficiency syndrome (AIDS) presents with the right fundus shown in the figure.
Which of the following statements regarding the patient's condition is most accurate?

The lesions may represent either cotton-wool spots or early CMV retinitis. Close observation of the patient for change
in these lesions, with documentation of the fundus by photographs, is indicated.
The lesions most likely are cotton-wool spots, which are a manifestation of human immunodeficiency virus (HIV)
related noninfectious retinal vasculopathy, and do not require close followup.

If the patient is without symptoms, and no retinal hemorrhage is associated with these lesions, they do not represent
CMV retinitis.

The lesions most likely represent early CMV retinitis if the patient's CD4 lymphocyte count is less than 50 cells per
cubic mm.
Please select an answer
Feedback: These white retinal lesions may represent either cotton-wool spots, a manifestation of HIV-related noninfectious
retinal vasculopathy, or the earliest manifestation of CMV retinitis. Both the retinal vasculopathy associated with HIV
infections and CMV retinitis are typically late manifestations of AIDS. CMV retinitis, in particular, is more common when CD4
lymphocyte counts are reduced to 50cells/cubic mm or less. CMV retinitis often may present without symptoms, and
although the retinal lesions are characteristically areas of hemorrhagic retinal necrosis, atypical presentations of CMV
retinitis may occur. Therefore, the safest course in this case is to document the fundus appearance with photographs, and
observe closely for any enlargement or change in these lesions, which would indicate CMV retinitis. Close observation of the
patient for change in these lesions, with documentation of the fundus by photographs, is indicated.

Question 76 of 130
A 30-year-old woman complains of blurred central vision in the right eye for the past 3 days. Multiple chorioretinal lesions
are noted in both eyes. Which of the following findings is most consistent with a diagnosis of acute posterior multifocal
placoid pigment epitheliopathy (APMPPE)?

Clear vitreous and subtle disc edema

Subretinal hemorrhage in the macula and peripheral scotomas

Vitreous cells and satellite lesions

Symptoms in the other eye within 2 days


Please select an answer
Feedback: Possible diagnoses, besides APMPPE, are presumed ocular histoplasmosis syndrome (POHS), recurrent
multifocal choroiditis (pseudo POHS), and toxoplasmosis. While all of these conditions have quite different clinical findings
and presentations, variants of them can sometimes be confusing. The typical POHS case has peripapillary atrophy of the
retinal pigment epithelium and peripheral "histo spots"; and, in the macula, choroidal neovascularization with subretinal fluid
and hemorrhage can occur. Bilaterality is common, but usually years elapse between macular involvement of the first eye
and that of the other eye. Vitreous cells are not seen in typical POHS. Recurrent multifocal choroiditis mimics POHS. It is
distinguished clinically from POHS by mild panuveitis and frequent recurrences, with the development of new fundus
lesions. Unilateral cases may become bilateral. Toxoplasmosis produces focal areas of chorioretinal inflammation, often
adjacent to previous areas of involvement (satellite lesions). Heavy vitreous cellular infiltration is the rule, and it is unusual to
have simultaneous bilateral involvement. APMPPE usually occurs following a viral illness and is usually bilateral, with rapid
involvement of both eyes once symptoms begin. Spontaneous recovery of vision is usual.
Question 77 of 130
Which of the following statements does not accurately describe sympathetic ophthalmia?

High-dose systemic corticosteroids are the initial treatment of choice in sympathetic ophthalmia.

Phacoanaphylaxis has been noted in up to 25% of patients with sympathetic ophthalmia.

Sympathetic ophthalmia may be differentiated from Vogt-Koyanagi-Harada syndrome by the absence of vitiligo and
alopecia in the former.

Sympathetic ophthalmia rarely occurs within 2 weeks of injury.


Please select an answer
Feedback: Sympathetic ophthalmia is a rare, bilateral, granulomatous uveitis that may occur following ocular trauma or
surgery. The onset of inflammation rarely occurs within 2 weeks of injury and may not occur until decades after trauma. The
majority (90%) of cases, however, begin within 1 year. Symptoms in sympathetic ophthalmia typically include pain,
photophobia, blurred vision, and decreased accommodation. Examination may reveal keratitic precipitates on the cornea,
anterior uveitis, vitritis, retinal edema, papillitis, and the yellow-white subretinal pigment epithelium (RPE) Dalen-Fuchs
nodules. Fluorescein angiography typically shows multiple pinpoint areas of hyperfluorescence. Pathologic examination of
eyes with sympathetic ophthalmia has shown a frequent association of phacoanaphylaxis (up to 25%). Sympathetic
ophthalmia may be difficult to distinguish clinically from Vogt-Koyanagi-Harada (VKH) syndrome, because they share not
only similar ocular findings, but also systemic findings such as vitiligo and alopecia. Vitiligo and alopecia, which are typical in
VKH, may also be seen in sympathetic ophthalmia, although less commonly. Debate still exists as to the benefit of
enucleating the exciting eye once sympathetic ophthalmia has begun. However, high-dose systemic corticosteroids (100-
200 mg) have been shown to be effective in the initial treatment of sympathetic ophthalmia. In patients unable to tolerate
systemic corticosteroids, immunosuppressive agents such as methotrexate, azathioprine, or cyclosporin may be used.

Question 78 of 130
Choroidal hemangiomas are not often associated with which of the following findings?

A subretinal neovascular membrane

Subretinal serous or lipid exudation

High internal reflectivity on A-scan echography

Leakage of dye in the late phases of fluorescein angiography


Please select an answer
Feedback: Choroidal hemangiomas are benign vascular tumors that may arise as isolated lesions without other vascular
malformations, as diffuse thickening of the choroid in association with Sturge-Weber syndrome, or a nonfamilial condition
characterized by ipsilateral angiomatous lesions involving the face, brain, and uveal tract. In contrast to melanomas,
choroidal hemangiomas exhibit high internal reflectivity on A-scan echography. During fluorescein angiography, they fill
during the prearteriole and arteriole phases and leak during the late phases. Serous or lipid exudation accounts for loss of
vision in many patients. Subretinal neovascular membranes rarely, if ever, develop from choroidal hemangiomas.
Question 79 of 130
A-26 year-old Caucasian woman with bilateral intermediate uveitis of two years duration complains of shooting pains down
her leg after a hot bath. What is the most likely diagnosis?

multiple sclerosis

Behcets disease

B-cell lymphoma

toxoplasmosis
Please select an answer
Feedback: This symptom is Lehrmitte’s sign, which is found in patients with multiple sclerosis. The two most common
underlying entities associated with intermediate uveitis are multiple sclerosis and sarcoidosis. The other entities can be
considered, but are unlikely in this patient.

Question 80 of 130
A 27-year-old woman in good health presents with a 6-week history of floaters OU, and a 3-week history of blurred visual
acuity OD. The fundus findings in the right eye are shown in the figures (posterior pole, A; inferior temporal periphery, B).
Examination of the left eye was significant for anterior vitreous cells, and a small pars plana snowbank. What is the most
likely diagnosis in this patient?
Pars planitis with peripheral retinal angioma

Coats' retinopathy

Cavernous hemangioma of the retina

Retinal angiomatosis with von Hippel-Lindau disease


Please select an answer
Feedback: The fundus findings in the right eye are most consistent with a peripheral retinal angioma (capillary
hemangioma) and exudative retinal detachment, possibly secondary to peripheral uveitis or pars planitis. The symptoms of
floaters OU and vitreous cells with pars plana exudate OS in a young patient are suggestive for peripheral uveitis. However,
other inflammatory causes such as sarcoidosis and Lyme disease should be ruled out with laboratory testing. Peripheral
retinal angiomas are a rare complication of peripheral uveitis and have also been seen in retinitis pigmentosa, after retinal
surgery, and as isolated lesions. They are best known in association with von Hippel-Lindau disease, where these angiomas
typically have a large feeding arteriole and venule, which is lacking in this patient. The lesions in von hippel-Lindau disease
are also often multiple and bilateral. Coats' retinopathy can also produce an exudative retinal detachment, but this would be
typically uilateral and unusual in a woman. Signs of inflammation would also not be expected in the fellow eye in Coats'
retinopathy. A cavernous hemangioma of the retina typically has a different appearance from the one seen in this patient,
with the lesion consisting of saccular aneurysms typically having the appearance of a cluster of grapes. Also, cavernous
hemangiomas of the retina are not associated with retinal exudation, which is prominent in this patient.
Question 81 of 130
A patient has bilateral diffuse non-necrotizing anterior scleritis. His vision is 20/20 OU and he has normal intraocular
pressure. He has no intraocular inflammation, and his evaluation for a systemic disorder is not revealing. Which of the
following interventions is the preferred first line therapy?

Flucinolone implant

Intraveous cyclophosphamide

Oral nonsteroidal anti-inflammatory drugs

Topical steroids
Please select an answer
Feedback: Oral nonsteroidal anti-inflammatory drugs are effective first line therapy for minimally symptomatic scleritis. If
these agents fail to suppress the inflammation, oral corticosteroids may be considered. Subtenons triamcinolone (STK) can
be used to treat non-necrotic scleritis, however, because of the risk of inducing a scleral melt, STK use is controversial.
Topical corticosteroids have littel effect on scleritis. A flucinolone implant is effective for posterior uveitis, but is not useful for
scleritis. Cyclosporin can be administered orally or topically, but not given intravenously for ocular disease.

Question 82 of 130
Which of the following patients would have the highest likelihood of developing posterior capsular opacification following
uncomplicated cataract extraction with posterior chamber lens implantation?

A 35-year-old man with intermediate uveitis

A 67-year-old man with recurrent toxoplasmosis

A 61-year-old man with a traumatic cataract

A 58-year-old woman with rheumatoid arthritis


Please select an answer
Feedback: Intermediate uveitis is associated with the development of posterior capsular cararact, and following cataract
removal an increased incidenct of opacification of the posterior capsule. Chorioretinitis from toxoplasmosis is not associated
with cataract or posterior capsular opacification. Oral steroid therapy for rheumatoid arthritis may lead to posterior capsular
cataract. Traumatic cataract may result in capsular or zonular weakness, or posterior capsular fibrosis, but does not
increase the likelihood of post-surgical development of posterior capsular opacification.

Question 83 of 130
A 33-year-old male presents with a first episode of moderate (2+) anterior chamber cells and flare, and 3 clock hours of
posterior synechiae. Ophthalmoscopy demonstrates 2+ vitreous haze and scattered, chorioretinal lesions. How would this
uveitis be characterized?

Panuveitis

Intermediate and posterior uveitis

Anterior and posterior uveitis

Posterior uveitis
Please select an answer
Feedback: The presence of uniform severity of inflammation of the anterior segment, vitreous, and choroid would qualify as
a panuveitis. In cases in which inflammation of one site, such as the posterior segment, result in "spill-over" inflammation in
other ocular compartments or tissues, a panuveitis may be mis-classified.

Question 84 of 130
A 34-year-old man presents with floaters and decreased vision in the left eye. He denies any extra-ocular disorder. His right
eye examination is normal. The left eye visual acuity measures 20/40, he has 1-2+ vitreous cells, and 3 areas of
circumferentially oriented retinal whitening in the midretinal periphery. What is the most likely diagnosis?

Acute retinal necrosis (ARN)

Cytomegallovirus (CMV) retinopathy

Lymphoma

Ocular toxoplasmosis
Please select an answer
Feedback: The most likely diagnosis, given this constellation of findings in an otherwise healthy patient, is acute retinal
necrosis (ARN). All of the alternative diagnoses given can mimic some features of ARN. CMV develops in
immunosuppressed hosts, such as those with AIDS or prior organ or bone-marrow transplant. In the immunosuppressed
host, anterior chamber and vitreous cellular reaction is minimal or absent. Toxoplasmosis presents with a single region of
chorioretinitis, unless it complicates an immunosuppressed host. Lymphoma is rare in this age group.

Question 85 of 130
What is the most common ocular complication of rheumatoid arthritis?

Episcleritis

Keratoconjunctivitis sicca

Scleritis

Recurrent anterior uveitis


Please select an answer
Feedback: Dry eye, in the form of keratoconjunctivitis sicca is by far the most common complication of rheumatoid arthritis,
and it can have devastating consequences to the ocular surface and visual function. Scleritis and peripheral ulcerative
keratitis may destroy the eye if not treated aggressively. Their presence is indicative of systemic disease, but these findings
are not as common as k. sicca. Anterior uveitis is usually seen associated with the presence of scleritis or other
inflammatory findings.
Question 86 of 130
Which of the following initial therapies would be preferred for macula or optic nerve threatening acute retinal necrosis?

Oral acyclovir

Intravenous acyclovir

Intravenous ganciclovir

Oral valacyclovir
Please select an answer
Feedback: Intravenous acyclovir reaches near immediate virostatic levels and has excellent ocular penetration. Many
uveitis specialists now also advise adjunctive intravitreal antiviral (foscarnet or ganciclovir). Oral valacyclovir achieves high
serum levels of drug, but the delay to achieve therapeutic levels for macular or optic nerve threatening ARN may be
unacceptable. Oral acyclovir is irregularly absorbed, but should not be relied upon for immenently threatening disease.
Ganciclovir has activity against herpes simplex and zoster, but is not a first line treatment.

Question 87 of 130
A 37-year-old man with prior anterior uveitis has progressively increased inflammation each time steroid eyedrops are
stopped. He had an exacerbation a week ago, and the inflammation has largely resolved on a tapering schedule of topical
corticosteroids. He has developed a posterior subcapsular cataract and posterior synechiae right eye. Because of recently
reduced vision to 20/80 he wants cataract surgery as soon as possible. What approach to cataract surgery would be
preferred?

Delay cataract surgery until regimen stable for 3 months

Begin pre-operative oral corticosteroids 3 days prior to prompt cataract surgery

Initate methotrexate 1 month before prompt cataract surgery

Perform prompt cataract surgery and treat eny post-operative inflammation with aggresive topical corticosteroid
Please select an answer
Feedback: The widely accepted recommendation is to control the intraocular inflammation for at least 3 months before
performing elective surgery. Achieving suppression of uveitis may require immunosuppressives such as methotrexate, but
these typically require months to reach their maximum and stable level of effectiveness. In cases in which inflammation
cannot be suppressed to anterior chamber accelularity, perioperative corticosteroids can be considered as adjunctive
therapy, but with increased risk of perioperative uveitis exacerbation.

Question 88 of 130
What laboratory test has the highest predictive value for acute Bartonella burgdoferi infection?

Purified protein derivative

Fluorescent treponemal antibody absorption test (FTA-ABS)

Indirect immunofluorescence assay for Bartonella immunoglobulin G

Enzyme-linked immunosorbent assay for Bartonella immunoglobulin M (IgM)

Please select an answer


Feedback: The ELISA for Bartonella IgA has the highest predictive value for detecting acute Bartonella burgdorfi
infection. The most specific test for past Bartonell burgorfi is the indirect immunoflurescent assay for Bartonella IgG. The
FTA-ABS is positive for Bartonella, but is also positive for other treponemal diseases, such as syphilis. The purified protein
derivative, or Mantoux test, detects whether the host has delayed type hypersensistivity to tuberculosis or non-tuberculous
mycobacteria.

Question 89 of 130
Cyclosporine is thought to reduce chronic noninfectious intraocular inflammation by reducing the activity of which cell type?

Macrophages

B-lymphocytes

Langerhans cells

T-lymphocytes
Please select an answer
Feedback: Cyclosporing primarily affects T-lymphocytes. Rituximab has an antibody to an antigen expressed on B-
lymphocytes, but may be effective in diseases in which T-lymphocytes play a role. Langerhans cells are antigen-presenting
cells in the skin and ocular surface that are involved in initiating the inflammatory response. Macrophages are involved in
execution of the inflammatory arc, but they are not directly modulated by cyclosporin

Question 90 of 130
A patient with deep ocular pain and subretinal fluid is suspected of having posterior scleritis with an exudative retinal
detachment. What finding would confirm a diagnosis of posterior scleritis?

Vitritis

Vitreous hemorrhage

Cystoid macular edema

A "T" sign on B-scan ultrasound


Please select an answer
Feedback: Posterior scleritis may be difficult to diagnose because of its occult location. Although posterior scleritis can be
associated with intraocular inflammation, vitritis, and cystoid macular edema, such findings are non-specific. Fluid exudation
between the sclera and posterior Tenon's capsule near the optic nerve, detectable by echography or MRI, may show the
highly characteristic "T-sign".
Question 91 of 130

What form of uveitis is most likely to follow gastroenteritis from Salmonella contaminated food or a Chlamydial urethritis?

Birdshot retinochoroiditis

Serpiginous retinochoroiditis

Diffuse unilateral subacute neuroretinitis

Acute anterior uveitis


Please select an answer
Feedback:

HLA-B27-associated acute anterior uveitis has followed bacterial infections, particularly gram negative rods (and chlamydia)
of mucosal surfaces. It is not an active infection, but most likely an immune response stimulated by the mucosal infection.
Although this is fairly well established, in most patients with HLA-B27 uveitis, no such previous infection is known to have
occurred. In fact, while the association is well established, most patients with acute anterior uveitis have no history of
infection.

In rare cases a gastrointestinal bacterial infection could lead to perforation or a para-colinic abcess; in such cases metastatic
endophthalmitis must be considered. Similarly if there is severe or recurrent gastrointestinal disease, Whipple's disease or
inflammatory bowel disease must be considered.

Although many forms of uveitis may be precipitated by infection or in fact be infectious, in most cases, a specific association
has not been proven. Diffuse unilateral subacute neuroretinitis is due to a nematode. Serpiginous and birdshot have been
thought to be precipitated by infections, possibly viral, but this has not been established, and treatment with
immunosuppressive agents appears to be effective, making active infection unlikely.

Question 92 of 130
In multifocal choroiditis and panuveitis, which clinical finding would most strongly influence you to recommendation chronic
systemic immunosuppression?

Progressive subretinal fibrosis

Anterior chamber inflammation

Cystoid macular edema

Choroidal neovascularization
Please select an answer
Feedback: Subretinal fibrosis is a very difficult and potentially blinding complication of posterior uveitis. If there is ongoing
inflammation in the setting of progressive fibrosis, then aggressive therapy must be considered, although indeed it is unclear
whether it is effective it is in all cases. It is important to determine whether choroidal neovascular membrane is present,
which may need more urgent therapy than the slow onset of most immunosuppressive therapy (including possibly anti-
VEGF therapies). It is not established whether intravitreal corticosteroids, such as a flucinolone implant, may be an
alternative once control is obtained.Both anterior uveitis and cystoid macular edema may require chronic
immunosuppressive therapy, or may be controlled by other intervention, but are not the strongest indications for such
therapy in this disease.
Question 93 of 130
A 29-year-old caucasian woman complains of new floaters in both eyes. She has been camping in the eastern United States
and does not remember any tick bites, but does recall a red rash on one leg followed by bumps on her shins. She denies
any sensory or motor neurologic symptoms. In both eyes she has vitritis without any retinal or choroidal lesions. Her chest x-
ray and serum angiotensin converting enzyme (ACE) are normal. What is the preferred working diagnosis?

Multiple sclerosis-associated uveitis

Systemic lupus erythematosus

Sarcoidosis

Lyme disease
Please select an answer
Feedback: Given that the patient went camping in a Lyme endemic area and had a rash and symptoms of erythema
nodosum, the diagnosis of Lyme disease is highly likely. However, ACE and chest x-ray are fair screening tests for
sarcoidosis, which lowers this the likelihood of this disorder. However, if Lyme titers are negative, additional testing for
sarcoidosis should be pursued. Demyelinating disease is unlikely. With this constallation of signs and symptoms, Lyme
disease is more likely. Systemic lupus erythematosis is unlikely to present in this manner.

Question 94 of 130
A 32-year-old man with necrotizing scleritis and a history of recent onset sinus disease presents with necrotizing scleritis. On
physical examination, there are no other systemic complaints or findings. What laboratory test is most likely to lead to a
specific diagnosis?

Serum angiotensin converting enzyme

Serum antineutrophilic cytoplasmic antibodies (ANCA)

Complete blood count

Serum rheumatoid factor and antinuclear antibodies


Please select an answer
Feedback: The history is most consistent with Wegeners granulomatosis, and ANCA testing is reasonably sensitive and
specific (though not absolutlely so) with this clinical picture to strongly suggest the diagnosis. This would be an unusual
presentation (without other systemic findings) for rheumatoid arthritis or systemic lupus erythematosis; further serum
rheumatoid factor and antinuclear antibodies are very nonsepcific, and by themselves do not make a diagnosis. A complete
blood count is very nonspecific (in partiuclar as this would not be expected to be a systemic lymphoma or leukemia). This is
not a typical presentation of sarcoidosis; in any case the angiotensin converting enzyme is neither very sensitive nor specific
for sarcoidosis. A chest x-ray may be abnormal (or normal) in either sarcoidosis or Wegeners granulomatosis.

Question 95 of 130
Which of the following is a known side effect of cyclosporine therapy?

Renal insufficiency

Cystoid macular edema

Glaucoma

Cataract
Please select an answer
Feedback: Renal insufficiency and systemic hypertension are probably the most common side effects of concern with
cyclosporine. Cataract and glaucoma and central serous choroidopathy can occur from corticosteroid use by any route;
cystoid macular edema is not a complication of corticosteroids but is not known to be a side effect of cyclosporin.

Question 96 of 130
A 36-year-old African-American man presents with uveitis in his left eye. Slit lamp examination demonstrates 2+ anterior
chamber cells with 1+ flare and 2 posterior synechiae in the left eye. Numerous anterior vitreous cells also are present just
behind the lens. By ophthalmoscopy, no chorioretinal scarring is present. The right eye also has 2 posterior synechiae, but
no anterior chamber cells or flare, and there are no anterior vitreous cells or posterior segment abnormalities. How would
you classify this uveities episode?

Anterior uveitis

Anterior and intermediate uveitis

Panuveitis

Intermediate uveitis
Please select an answer
Feedback: Classifying the anatomic location of uveitis depends upon identifying the principal tissue affected. Usually this
corresponds with the site of greatest inflammation. The presence of "spill-over" cells in the anterior vitreous does not qualify
as intermediate uveitis, and would potentially confound formation of an appropriate differential diagnosis. Similarly, anatomic
complications such a cystoid macular edema do not change the classification. In this case the patient has clear evidence of
anterior but not intermediate or panuveitis.

Question 97 of 130
A 32-year-old Hispanic woman with prior multifocal choroiditis and panuveitis presents with a sudden onset of vision loss
OD. She has been off any therapy for 4 years. On examination, visual acuity is 20/200 OD with no anterior chamber
inflammation or vitritis. What findings would you expect on a fluorescein angiogram?

Early hyperfluorescence in the macula with late decreasing hyperfluorescence

Late leakage of the optic nerve

Early blockage and late staining in the macula

Early hyperfluorescence with late leakage in the macula


Please select an answer
Feedback: The most common reason for sudden vision loss in multifocal choroiditis with panuveitis is a choroidal
neovascular membrane. Such subretinal neovascular membranes also can be seen in punctate inner choroidopathy and
less commonly in birdshot chorioretinopathy. If there were active inflammation, more gradual vision loss, perhaps even to
20/200, could be from macular edema, but that would cause a different pattern of petaloid leakage gradually increasing on
angiography. Ocular coherence tomography may also be helpful. Late leakage of the optic nerve head would not be
expected as the optic nerve is not typically involved, although with sufficient active inflammation a papillitis is possible. Early
hyperfluorescence with late decreasing hyperfluorescence may be seen if there is a window defect from retinal pigment
epithelial changes that can be seen in this disease but this would not explain sudden vision loss. Early blockage and late
staining is more characteristic of other entities such as acute posterior multifocal placoid pigment epitheliopathy.
Question 98 of 130
What disorder is best characterized by a unilateral, mild, anterior uveitis with a sentinel keratic precipitate (KP) and high
intraocular pressure (IOP).

Juvenile idiopathic arthritis-associated anterior uveitis

Glaucomacyclitic crisis (Posner-Schlossman syndrome)

Sarcoidosis

Fuch's heterochromic iridocyclitis


Please select an answer
Feedback: Increased intraocular pressure in anterior uveitis can be secondary to chronic inflammatory damage to angle
structures, iris bombe, or corticosteroids, however, in the setting of mild uveitis and increased pressure at the beginning of
an episode, viral anterior uveitis must be considered. Glaucomacyclitic crisis is a self-limited, mild, unilateral anterior uveitis
with increased intraocular pressure, not secondary to corticosteroids. Viruses have been implicated, although due to the
self-limited nature of the process and the facts that CMV may be the virus involved, at least in some cases, and that oral
ganciclovir is not as well tolerated as acyclovir, antiviral therapy is not often used. Fuchs heterchromic idirdocycloitis is a
chronic anterior (and sometimes intermediate) uveitis. There are diffuse stellate keratic precipitates. The uveitis with juvenile
idiopathic arthritis is chronic and bilateral. Sarcoidosis is rarely if ever associated with a mild recurrent anterior self limited
uveitis with increased intra-ocular pressure without corticosteroid use.

Question 99 of 130
High dose corticosteroids have controlled the necrotizing scleritis associated with Wegener's granulomatosis in your patient.
What would be the preferred second intervention?

Add a systemic immunosuprressive agent such as cyclophosphamide, infliximab, or mycophenolate.

Continue high-dose corticosteroids (20 mg/day) for one year and then add other agents as needed.

Add an oral nonsteroidal anti-inflamatory agent such as indomethacin.

Add intensive topical corticosteroid and nonsteroidal anti-inflammatory agents.


Please select an answer
Feedback: In necrotizing scleritis associated with systemic disease, topical therapy or oral nonsteroidal inflammatory agents
would be insufficient. Indomethacin and related medications are appropriate for mild, non-necrotizing scleritis and may help
the pain, but is insufficient treatment for necrotizing scleritis. Topical agents are also insufficient therapy (and some would
consider relatively contra-indicated, especially if any corneal involvement).They also will not treat the associated systemic
disease. Long-term high-dose corticosteroids are not recommended as sole therapy because of significant side effects.
Cytotoxic agents have long been shown to be life saving in Wegeners granulomatosis, although now milder forms are
recognized and other agents such as mycophenolate, infliximab and rituximab may be effective.
Question 100 of 130
A 28-year-old Caucasian woman with intermediate uveitis complains of paresthesisas after a hot shower. She is from the
Midwest. What disorder should be investigated as a likely cause?

Whipple's disease

HLA-B27-associated uveits

Behcets disease

Demyelinating disease
Please select an answer
Feedback: There is an association between intermediate uveitis and multiple sclerosis; it is not rare. One can have uveitis
and neuropathies with Behcets disease, but this would be very rare, and longstanding intermediate uveitis would be a very
atypical form of uveitis in Behcets disease. Whipple's disease might be considered as there are many manifestations, which
could include intermediate uveitis and neurologic involvement, but it is very rare and much less likely given the description of
Lehrmitte's sign in this patient. Intermediate uveitis, except secondary to severe anterior uveitis, is not usually an HLA-B27-
associated form of uveitis.

Question 101 of 130


A male patient with uveitis and a history of unprotected sex with prostitutes is suspected of having syphilitic uveitis. His rapid
plasma reagin (RPR) testing syphilis serology is negative. What is the next step in his evaluation?

Obtain a fluorescent treponemal antibody absorption (FTA-ABS) test.

Treat with penicillin.

Do blood cultures for syphilis.

Do a paracentesis for darkfield microscopy.

Please select an answer


Feedback:

A false-positive RPR is possible in late syphilis, which uveitis is likely to be. A FTA-ABS is more likely to be positive, with
false negatives being very rare. Other treponemal absorption tests, such as the microhemagglutination-Treponema pallidum
(MHA-TP), are also useful. One problem with the FTA-ABS is false positives, especially if weakly reactive. Although treating
with penicillin may be reasonable, and severe and fatal reactions are rare, the best next step is to try to diagnose. Blood
cultures are not feasible for diagnosing syphilis. Availability of darkfield microscopy is very limited, it requires great expertise,
it can result in a false negative, and although a positive result would clinch the presence of spirochetes, it would not
necessarily confirm syphilis.
Question 102 of 130
What is the most common cause of visual loss in patients with birdshot chorioretinopathy?

Retinal detachment

Macular edema

Choroidal neovascularization

Retinal neovascularization
Please select an answer
Feedback: Cystoid macular edema is a common cause of decreased vision in birdshot chorioretinopathy. It is the most
common cause of mild and severe loss of Snellen visual acuity, although retinal atrophy late in the disease can also cause
severe visual disability. Retinal neovascularization is very rare, and unless there is vitroretinal traction or hemorrhage, rarely
causes vision loss. Choroidal neovascularization is also much less common than macular edema, but can cause severe loss
of central vision in birdshot chorioretinopathy.

Question 103 of 130


For the past year, a patient with iritis (anterior uveitis) in both eyes has recurrent episodes charaterized by 2-3 + anterior
chamber cells every time prednisolone actetate 1% drops are reduced fewer than 6 times per day. She has modest pain,
redness, and photophobia associated with episodes. Even at prednisolone acetate at 4 times per day, she has 1+ cells. She
has 4 clock hours of posterior synechiae and keratic precipitates (KP) in Alt's triangle that resolve when the corticosteroids
are increased. The KP are white and not spidery or spindly. She has a visually significant cataract and is anxious to pursue
cataract surgery right away. The patient does not have posterior uveitis or cystoid macular edema. What is the next step in
their management?

Methotrexate and intensive topical corticosteroids in the perioperative cataract surgery period

Intravitreal fluocinolone implant

Increase the prednisolone actetate 1% to every hour in the cataract surgery perioperative period

Immunomodulatory systemic agent

Please select an answer


Feedback:

The standard is to maintain good control of the inflammation for 3 months before pursuing surgery. It is perilous to do
otherwise unless the uveitis is very mild and without sequelae, and certainly would be appropriate for this patient who has
chronic disease with recurrences whenever she decreases the topical medications to a reasonable, sustainable level.
Recurrences of uveitis or chronic active uveitis can cause great damage, and so it is important to make sure the
inflammation is well controlled on a regimen the patinet is tolerating before surgery. Inflammation after cataract surgery can
result in a devastating and presistent cystoid macular edema in uveitis patients, or membrane formation with traction on the
retina or cilliary body with damage to those tisses and complications such as a retinal detachment or hypotony. If a lens
implant is placed there can be "cocooning" of the lens implant with membranes and lens implant displacement causing
glaucoma and corneal decompensation and traction on the ciliary body leading to hypotony and even phthisis. If the
inflammation is severe or persistant precipitates on the lens implant that can be visually significant. This patient is unlikely to
maintain long-term control or take hourly drops for many months without systemic immundulatory therapy such as
methotrexate. However, methotrexate takes more than 2 weeks to get the full effect, and it is important to be certain that the
patient will tolerate the medication long-term, therefore giving it for only 2 weeks and mainataining control short-term with
topical corticosteroids is insufficent. The fluocinolone implant is not approved for anterior uveitis. Also it is a relatively low
dose of corticosteroid and would take a while to be effective; meanwhile damage can occur. Such an approach is feasible if
there is chronic cystoid macular edema and that is the indication for the implant. Indeed the anterior uveitis may respond
over time, but again, any such response will take time. The symptoms and presence of posterior synechiae, the nature of
the keratic precipitates, and the response to coriticosteroids indicate that this is not Fuchs heterochromic iricocyclitis (FHI),
which can be bilateral. This is important because FHI does not require such aggressive preoperative therapy.

Question 104 of 130


For an otherwise healthy 30-year-old with no history of trauma, surgery, or intavenous drug use what agent most commonly
causes acute retinal necrosis (ARN)?

Toxoplasmosis infection

Cytomegalovirus (CMV) infection

Herpes zoster and herpes simplex virus infection

Lymphoma
Please select an answer
Feedback: Intraocular infection with herpes simplex or zoster are by far the most common causes of ARN in an otherwise
healthy individual. ARN is defined as a peripheral, rapidly progressive retinal necrosis with an inflammatory response. There
can also be posterior areas of involvement, and even the optic nerve may be involved, but the peripheral lesions are most
characteristic. CMV retinitis causes retinal necrosis, but without as much inflammation, and does not progress as rapidly as
ARN. CMV retinitis is typically found in immunosuppressed patients. Toxoplasmosis causes retinitis, but unless the host is
immunosuppressed retinal involvement is not widespread and rapidly progressive. Lymphoma can present clinically like a
retinitis, but it is not as rapidly progressive as ARN and is much less common, especially in this age group, than ARN
secondary to herpes zoster or simplex. However, it is reasonable to consider these and other infections or inflammatory
processes, especially if the process progresses despite adequate antiviral therapy.

Question 105 of 130


What is the most common cause of severe visual loss in patients with punctate inner choroiditis (PIC)?

Choroidal neovascularization

Macular edema

Retinal vasculitis

Severe vitritis

Please select an answer


Feedback: PIC is characterized by small (100-300 micromolar) chorioretinal lesions without overt inflammation. Similar
lesions can be seen in multifocal choroiditis (MFC) and panuveitis, and it may be that PIC is a variation of MFC. Similar
lesions can be seen in the periphery of the posterior segment in granulomatous processes such as sarcoidosis and familial
granulomatosis and in later phases of Vogt-Koyanagi-Harada disease. It also may resmble presumed ocular histoplasmosis
syndrome. It may be asymptomatic, and as there may be no overt inflammation, vision loss from severe vitritis, macular
edema, or retinal vasculitis is not typical of PIC. Choroidal neovascular membranes can occur and can result in vision loss.
Question 106 of 130
A 17-year-old girl complains of blurred vision and floaters which have increased over several months. Her visual acuity last
year was 20/20 OU, but now is 20/60 OD, 20/50 OS. Examination shows bilateral anterior chamber cells, a mild peripheral
cortical cataract, 1-2+ vitreous cells and haze, a few snowball vitreous opacities, and small snowbanks at the ora serrata
OU. What is the most likely cause of her decreased vision?

Vitritis

Peripheral retinal vasculitis

Macular edema

Cortical cataract

Please select an answer


Feedback: Macular edema is a common cause of decreased vision in intermediate uveitis. The cataract and vitritis
described are unlikely to cause a decrease in vision to 20/50 or less. Peripheral retinal vasculitis may be seen in
intermediate uveitis, but would not directly affect visual acuity, ie, without neovascular changes leading to vitreoretinal
traction or vitreous hemorrhage.

Question 107 of 130


What is the most definitive way to diagnose phacoantigenic (phacoanaphylactic) uveitis?

Lack of response to aggressive topical steroid treatment

Ultrabiomicroscopy (UBM)

Anterior chamber paracentesis

Clinical history
Please select an answer
Feedback: Although a history of recent trauma can suggest a likelihood of phacoantigenic uveitis, the most definitive way to
confirm the diagnosis is to examine intracular fluid for the characteristic macrophages with engulfed lens material.
Additionally, cultures may be performed to rule out infection. Lens-induced uveitis has been most associated with trauma or
retained lens fragments after surgery, but also may be associated with sympathetic ophtalmia and hypermature lenses.
Further, severe uveitis with an advanced cataract may make such a clinical distinction difficult to make, and infections can
mimic lens-induced uveitis in the context of trauma or surgery and also be poorly responsive to topical corticosteroids. UBM
can be suggested by showing retained lens fragments but is not definitive and will not rule out infection. It should be noted
that if urgent surgery is planned for a disrupted lens capusle with retained lens material, whether from surgery or injury,
material for histology and culture can be obtained at the time of surgery and surgery need not be delayed.
Question 108 of 130
A patient with scleritis is found to have a high titer of antineutrophil cytoplasmic antibodies (ANCA) with a cytoplasmic
pattern (c-ANCA). What is the most likely systemic disease associated with these findings?

Polyarteritis nodosa

Systemic lupus erythematosis

Wegener's granulomatosis

Rheumatoid arthritis

Please select an answer


Feedback: ANCA testing can be positive in uveitis without being specific for Wegener's granulomatosis. Polyarterisis
nodosa, rheumatoid arthritis, and systemic lupus erythematosis are all associated with scleritis. However, in the context of a
positive c-ANCA test and scleritis, Weger's granulmotosis is the most likely. Although Wegener's granulomatosis may be
limited to the eye or orbit, it is a potentially fatal disease, and a prompt diagnosis and appropriate treatment may be both
vision- and life-saving.

Question 109 of 130


Progressive multifocal leukencephalopathy is associated with what virus?

Varicella zoster virus

Jacob-Cruzfeld virus

Human papilloma virus

Cytomegalovirus
Please select an answer
Feedback: Progressive multifocal leukoencepholopathy is a devastating central nervous system (CNS) infection. The virus
has been demonstrated in the ocualr tissues of AIDS patients and it has been associated with a multifocal retinitis in one
AIDS patient, but the main concern to opthtalmologists is that the CNS infection can be unmasked by immunosuppressive
agents such as mycophenolate used to treat uveitis. It is associated with the Jacob-Cruizfeld virus. Human papilloma virus
has not been reported to cause ophthalmic findings. Cytomegalovirus and varicella zoster cause ocular and CNS findings
but not progressive multifocal dystrophy.

Question 110 of 130


What term best characterizes intraocular inflammation predominantly involving the vitreous and peripheral retina?

Panuveitis

Multifocal choroiditis

Intermediate uveitis

Iridocyclitis
Please select an answer
Feedback: Uveitis is characterized by many factors including onset type, location and association with other inflammatory
conditions (disease or test result). Involvement of the vitreous and peripheral retina represents the middle portion of the
eye, compared to the anterior, posterior or entire eye (panuveitis). The term iridocyclitis refers to involvement of the iris and
ciliary body which does not apply to this patient. Multifocal choroiditis (MFC) is a descriptive term that may be associated
with a few vitreous cells but involvement of the vitreous is not a prominent feature. The choroidal inflammaiton would be
predominant with possibly secodnary retinal fondings such as "punched out" chorioretinal lesions. Panuveitis could be
considered if there were chorioretinal findings and anterior findings.

Question 111 of 130


A 45-year-old woman has Fuch's heterochromic iridocyclitis (FHI) has bilateral 1+ cells, 1+ flare, normal intraocular pressure
and minimal posterior subcapsular cataracts. Her vision is good and she has no visual complaints. What is the preferred
treatment?

Methotrexate

Topical corticosteroids

Observation, monitoring cataracts and intraocular pressure

Paracentesis for rubella virus


Please select an answer
Feedback: For patients that have minimal visual disability it is generally preferred to monitor patients with FHI for ocular
complications rather than treating mild inflammation. These eyes are rarely symptomatic from inflammation and do not form
posterior synechiae or develop cystoid macular edema. The risks of treatment usually outweigh the potential benefits. If
cataract surgery is contemplated, corticosteroids may be useful in the perioperative period. Although rubella and
cytomegalovirus (CMV), have been implicated in FHI, there is no treatment for rubella. Treatment for cytomegalovirus has
significant known side effects and would be relatively contraindicated in a benign disorder like FHI.

Question 112 of 130


What treatment is thought to prevent sympathetic ophthalmia?

Oral corticosteroids to treat traumatic uveitis

Immunosuppressive agents at the first sign of inflammation

Enucleation within 14 days of ocular injury

Removal of orbital foreign bodies


Please select an answer
Feedback: The only treatment generally thought to prevent sympathetic ophthalmia (SO) is removal of the injured eye prior
to systemic sensitization (accepted as 14 days). Although enucleation is usually performed, some believe evisceration may
also be effective. Medical therapy is unproven a a means to prevent SO, though has been used to successfully treat SO.
Removal of orbital antigens could possibly play a role. There is no evidence that removing a foreign body would reduce risk.
The standard of care is to remove an eye thought to have no visual capacity.
Question 113 of 130
The non-steroidal anti-inflammatory drug, celecoxib (Celebrex), may be useful in preventing recurrences of acute anterior
uveitis. What are the FDA-mandated black-box warnings regarding this agent?

Hypertension

Fluid retention and congestive heart failure

Potentially fatal cardiovascular thrombotic and gastrointestinal complications

Renal insufficiency

Please select an answer


Feedback: Although all these are potential problems with celecoxib, potentially fatal cardiovascular disorders (including
myocardial infarction and stroke) and gastrointestinal complications (perforation, hemorrhage) are deemed worthy of black-
box warnings. Celecoxib is tolerated without difficulty in most patients, but it is important always to judge the potential
benefits against the potential risks.

Question 114 of 130


For what type of uveitis would methotrexate immunosuppression be contraindicated?

Infectious uveitis

Pediatric uveitis

Sarcoid uveitis

Idiopathic uveitis
Please select an answer
Feedback: Methotrexate is commonly used as a corticosteroid-sparing agent in noninfectious uveitis. It is contraindicated in
infectious uveitis as the infection may be worsened

Question 115 of 130


A patient with systemic lupus erythematopsis (SLE) presents with cotton-wool spots on retinal examination. What test may
be helpful in evaluating her eyes for choroidal retinopathy?

Indocyanine green (ICG) angiography

Ocular coherence tomorgraphy (OCT)

Blood Pressure

Antinuclear antibody (ANA) testing for titers

Please select an answer


Feedback: ICG or fluorescein angiography would be the most likely tests to demonstrate lupus choroidal involvement.
However, only ICG is given as an option. OCT can provide information regarding choroidal thickness, but is not a functional
test. An elevated blood pressure could indicated an increased likelihood of hypertensive choroidopathy related to lupus, but
is not is a specfic test. Antinuclear antibody is useful when evaluating the differential, but does alone does not suggest
choroidal involvement.

Question 116 of 130


In a patient with birdshot chorioretinopathy (BCR) for 2 years, what is the most common cause of treatable decreased visual
acuity?

Cystoid macular edema (CME)

Retinal atrophy

Epiretinal membrane (ERM)

Subretinal neovascular membrane (SRNVM)


Please select an answer
Feedback: CME is the most common cause of decreased visual acuity in patients with BCR. Profound visual disfunction
may be from retinal dysfunction causing abnormal visual field testing, contrast and color and electrophysiology testing.
Central vision is affected later. When retinal atrophy, treatment is unlikely to be visually beneficial. SRNVM and ERM occur
in BCR and may respond to therapy, but are less common causes of decreased visual acuity than CME.

Question 117 of 130

For six years, a 30-year-old woman has had poorly-controlled bilateral intermediate uveitis without any systemic symptoms.
A chest x-ray and tuberculin skin-test is normal. She has not traveled internationally. What associated disorder would
constitute a contraindication to adding a tumor necrosis factor inhibitor treatment?

Lymphoma

Tuberculosis

Sarcoidosis

Demyelinating disease
Please select an answer
Feedback:

Intermediate uveitis is one presentation of sarcoidosis. TNF inhibitors have been reported to treat sarcoidosis and not
contraindicated. Tuberculosis would be a contraindication to TNF inhibitors but the CXR and tuberculin test are negative.
Lymphoma is extraordinarily rate in a 30-year-old, unless it is metastatic, for which there is no evidence. Demyelinating
disease (multiple sclerosis) has a known strong association with intermediate uveitis, and TNF inhibitors are relatively
contraindicated as they have been associated with optic neuritis and exacerbations of multiple sclerosis.
Question 118 of 130
An immunocompetent adult has a chronic bilateral anterior uveitis. What positive blood test would suggest a specific
diagnosis and therapy?

Serum antinuclear antibody (ANA) titers

Microhemagglutination assay for treponema pallidum (MHA-TP)

Serum angiotensin converting enzyme level

Human leukocyte antigen (HLA) testing


Please select an answer
Feedback:

Microhemagglutin assay (MHA-TP) or Fluorescent treponemal antibody-absorbed (FTA-ABS) are highly specific for syphilis.
The treatment would be systemic antibiotics, a specific treatment for syphilis. Any of the other tests would not be specific
for a disorder or a indicate a specific treatment. Immunosuppression is a non-specific therapy whether using corticosteroids
or immunosuppression. Serum ANA is rarely helpful in uveitis, and is more likely to be a false positive or indicate a disease
like systemic lupus which rarely if ever will cause an isolated bilateral anterior uveitis (as opposed to retinal or choroidal
vasculopathy or scleritis).a testing in of itself is not sensitive or specific enough to make a diagnosis with the possible
exception of HLA-A29 in birdshot chorioretinopathy, but that would not present with a bilateral anterior uveitis. The patient
could have sarcoidosis, but a serum angiotensin converting enzyme is at best a poor screening test with a very low
sensitivity and specificity so does not make the diagnosis.

Fluoresent ANA titers are more likely to be a false positive in this context. Bilateral chronic anterior uveitis is an unusual
presentation of systemic lupus erythematosis, and many individuals without systemic disease have positive ANA testing.
HLA testing may suggest a diagnosis, but is not specific or sensitive enough to suggest a particular therapy. For example,
the strongest HLA associations with uveitis are birdshot chorioretinopathy (HLA-A29), acute anterior uveitis and HLA-B27,
and tubulointerstitial nephritis and uveitis syndrome (which may be associated with chronic bilateral uveitis). In none of these
entities is the diagnosis made by HLA testing (although such testing may support the diagnosis) nor are therapeutic
decisions based on HLA testing. Although an angiotensin-converting enzyme may be useful, it is not highly specific or
sensitive for sarcoidosis and alone does not suggest a specific therapy. Postive antitreponemal testing suggests a specific
therapy: antibiotic treatment at doses sufficient to treat possible neurosyphilis, unless there is reason to do otherwise.

Question 119 of 130


What is the most common non-ocular finding in Adamantiades-Behcet disease?

Arthritis

Central nervous system vasculitis

Oral or genital ulcers

HLA-B51 positivity

Please select an answer


Feedback: There are many systemic findings in Adamantiades-Behcet disease, but the most common are oral and genital
ulcers. Systemic vascular involvement may be fatal and is certainly of concern, but is less common. HLA-B51 is associated
with Adamantiades-Behcet disease, but not so strong that it has a high positive and negative predictive value. Additional
ocular manifestations of Adamantiades-Behcet disease, including acute anterior uveitis may include hypopyon formation and
an occlusive retinal vasculitis that may be blinding.
Question 120 of 130
What ocular findings are most characteristic of the ocular involvement of West Nile virus?

Chronic bilateral granulomatous anterior uveitis

Chorioretinal lesions

Scleritis

Unilateral acute anterior uveitis with hypopion formation

Please select an answer


Feedback: West Nile virus infection is rarely associated with ocular disease, but when it is, the most characterisitc finding
are the horioretinal lesions. Conjunctivitis and nongranulomatous anterior uveitis may also be found.

Question 121 of 130


What micro-organism has been associated with Fuchs hetereochromic iridocyclitis?

A virus

A bacteria

A parasite

A fungus
Please select an answer
Feedback:

Recent epidemiologic studies and studies using polymerase chain techniques have suggested viruses, particularly rubella
and cytomegalovirus are associated with Fuch's heterochromic iridocyclitis. Previous speculation of an association with
toxoplasmosis has not been supported by PCR and was based primarily on the concurrence of inactive chorioretinal scars
and serologic positivity in some patients.
Question 122 of 130
When is it diagnostically useful to obtain an antinuclear antibody (ANA) test in a patient with isolated anterior uveitis and no
skin, joint, or other systemic problems?

Never

When the unilateral anterior uveitis is chronic

Always

When the bilateral anterior uveitis is acute

Please select an answer


Feedback: An ANA is usually obtained to evaluate the possibility of systemic lupus erythematosis (SLE). SLE is rarely if
ever associated with isolated anterior uveitis. In addition, many patients have positive ANA testing without systemic disease,
and the diagnosis of SLE is made on multiple criteria, not only ANA testing. If SLE is suspected, a systemic examination with
antibody testing including ANA is indicated. SLE is most commonly associated with scleritis or retinal vasculopathy if there
are intraocular complications of disease. Even then it is uncommon for SLE to present initially with ocular inflammation.
Retinal or choroidal vasculopathy raises concern of central nervous system involvement and requires urgent consultation
with internal medicine or rheumatology.

Question 123 of 130


What percentage of patients who present with unilateral, nongranulomatous, acute anterior uveitis (AAU) express the HLA-
B27 haplotype?

10%

95%

50%

20%
Please select an answer
Feedback: HLA-B27 positivity is found in about 8% of the Caucasian population, but in 50-65% of patients with unilateral
AAU. The diagnosis is not made by HLA testing, which has a poor predictive negativity and positivity, nor will finding that the
patient is HLA-B27 positive change management. Being HLA-B27 positive with uveitis is, however, associated with an
increased chance of spondyloarthropathy.
Question 124 of 130
A 34-year-old myopic woman has an asymmetric chronic uveitis characterized by periodic exacerbations, a mild anterior
uveitis, vitritis, and 50-200 micron atrophic (punched-out) choroidal lesions. Her chest CT scan is normal and her HLA-A29
negative. On a previous visit to the referring ophthalmologist, A fluorescein angiography (FA) of the chorioretinal lesions
showed early blockage and late staining. What is this woman's the most likely diagnosis?

Birdshot chorioretinopathy

Sarcoidosis

Multifocal choroiditis and panuveitis syndrome

Presumed ocular histoplasmosis syndrome (POHS)

Please select an answer


Feedback:

Although several entities may rarely present this way, the case description is most consistent with multifocal choroiditis and
panuveitis syndrome. POHS is unlikely because of the vitreous cells present. Sarcoidosis is unlikely as the the chest CT is
normal, the anterior uveitis is typically granulomatous, and some retinal vascular inflammation is present. Birdshot
chorioretinopathy is rare at this age, usually has a positive HLA-A29, and have discreet scattered lesions

Question 125 of 130


Over several weeks, an immunocompetent patient’s left eye developed 1+ anterior chamber cells, enlarging localized
corneal edema and keratic precipitates in a region of linear endothelial translucency. There is no vitritis, chorioretinal lesion
or evidence of posterior segment inflammation. The patient has not used intravenous drugs, has no systemic complaints,
injuries, or recent surgery. What category of infection is most likely causing the patient's problem?

Fungus

Parasite

Virus

Bacteria
Please select an answer
Feedback:

The case presented is of cytomegalovirus anterior uveitis and corneal endothelitis, but the features are similar to the
presentation of other herpes viruses. Anterior uveitis from viruses can be associated with diffuse or localized keratic
precipitates, corneal or iris change (usually atrophy rather than nodules), and increased intraocular pressure at the
onset of disease. Although metastatic infections are from bacteria or fungi are possible, they are rarer and usually
have an suggestive history or setting. Toxoplasmosis (or other parasites) as an isolated anterior uveitis would be very
rare in an immunocompetent patient, but has been reported in the setting of AIDS.
Question 126 of 130
A patient has chronic bilateral panuveitis that began several weeks after a penetrating injury to the right eye. Both eyes have
anterior chamber and vitreous cells, and macular edema despite hourly topical corticosteroid therapy and prednisone 15
mg/day. The inflammation was reduced by higher dose oral corticosteroids, but he cannot be weaned to less than 15
mg/day prednisone without an increase of inflammation. His visual acuity is 20/80 in the injured eye and 20/60 in the fellow
eye. What therapy would be preferred next?

Control the inflammation with corticosteroids, but start non-steroidal immunosuppressive therapy.

Eviscerate the injured eye.

Treat with long-term high-dose systemic corticosteroids.

Enucleate the injured eye.


Please select an answer
Feedback: Late enucleation and evisceration have not been shown to be beneficial for sympathetic ophthalmia, the cause
of this patients panuveitis. Given the functional visual acuity in the injured eye, enucleation or evisceration would be
inappropriate. Long-term high-dose corticosteroids are poorly tolerated, but may be reduced by initiating other
immunosuppressive agents or use of intraocular corticosteroid implants.

Question 127 of 130


Four months after cataract surgery, a patient presents with persistent iritis. The uveitis was present shortly after surgery
despite topical corticosteroids and has gradually worsened. There is prior eye disease. On examination the eye has mutton-
fat keratic precipitates, 3+ cells and flare, a capsular plaque and a mild vitritis. There are no chorioretinal inflammatory
lesions or residual lens material. There is a well-centered intraocular lens implant. What is the most likely diagnosis?

Sarcoidosis

Toxoplasmosis

P. acnes endophthalmitis

Uveitis-glaucoma-hyphema syndrome

Please select an answer


Feedback: Chronic inflammation following cataract surgery may result from many causes. Lens cortex or nucleus may be
retained even if the capsule is intact. However, the constellation of findings strongly implicates the diagnosis of P. acnes
endophthalmitis. Non-surgically related uveitis such as sarcoidosis may be precipitated by surgery or may have been
unrecognized preoperatively. Toxoplasmosis is usually associated with a retinochoroiditis (with very rare exceptions). Viral
uveitis may rarely present similarly to this case; in such cases PCR of intraocular fluids may be helpful. Uveitis-glaucoma-
hyphema symdrome is rare and may be difficult to diagnose. UGH is less likely with a well centered intraocular lens, except
in the setting of pseudoexfoliation.
Question 128 of 130
In chronic iridocyclitis, when would daily atropine 1% be indicated to prevent posterior synechiae?

In the setting of chronic flare and prior posterior synchiae

In the setting iris atrophy/transillumination defects

In the setting of visual loss from macular edema

Rarely

Please select an answer


Feedback:

Chronic anterior uveitis , by definition, persists longer than 3 months. Keeping the pupil immobile is likely to facilitate
synechiae formation long-term. The resulting synechiae typically occur in the dilated position. Less potent cycloplegia
allows some pupil movement, which may break up or prevent synechiae. In contrast, for acute anterior uveitis, atropine may
be helpful as the iris often responds poorly to cycloplegia. In this setting strong dilation may break up recently formed
posterior synechiae. Atropine might be helpful long term treatment of uveitis in unusual circumstances such as uveitis-
glaucoma-hyphema (UGH) syndrome or pigmentary dispersion syndromes. The presence of macular edema or iris
atrophy/transillumination defects does not indicate the use of atropine.

Question 129 of 130


A patient with Vogt-Koyanagi-Harada disease has supression of her chronic anterior uveitis on methotrexate. What is the
preferred minimum duration that supression should be maintained before cataract surgery is performed?

3 weeks

3 months

6 months

As soon as uveitis is supressed (no cells in the anterior chamber)


Please select an answer
Feedback: Three months is the preferred duration to suppress an anterior uveitis before performing elective surgery.
Because of the duration necessary for methotrexate to have its full effect (6 to 12 weeks) the duration from the time of
initiating methotrexate may be approximately 6 months.
Question 130 of 130
What systemic antiviral therapy has been reported to have hypotony and intraocular inflammation as side-effects?

Intravenous cidofovir

Intravenous foscarnet

Intravenous ganciclovir

Oral valganciclovir
Please select an answer
Feedback: IV Cidofovir has resulted in severe hypotony and anterior uveitis in as many as half of the patients treated for
CMV retinitis. The other medication have not been associated with either hypotony or inflammation.

Collected from www.aao.org (self assessment)


By Dr. AlBaraa AlQassimi

Вам также может понравиться